Вы находитесь на странице: 1из 66

BANKiq.

IN MAY 2019

INDEX
5 GROUP DISCUSSION TOPICS FOR 1
BANK EXAMS

TIP OF THE MONTH 1

BANKING TERMINOLOGY 1-2

ABBREVIATIONS 2

TOPIC IN NEWS 3-7

1) Banking and Financial Awareness 8-11


2) Taxation 12-13
3) Agriculture 14
4) Industry 15
5) Infrastructure 16
6) International Affairs 17
7) India and the World 18-21
8) Polity and Governance 22
9) Appointments and Resignations 23
10) Government Schemes 24
11) Miscellaneous 25-31

PRACTICE CURRENT AFFAIRS MCQ’S


1) Banking and Financial Awareness 32
2) Appointments & Resignations 33
3) Days & dates 34
4) Defence & Security 35
5) Honours and Awards 35-36
6) India and the World 37
7) Industry 37
8) Infrastructure 37
9) International Affairs 38
10) Polity and Governance 39
11) Reports and Indices 39
12) Sports 40-41
13) Miscellaneous 42-43

SOLUTIONS CURRENT AFFAIRS MCQ’s


1) Banking and Financial Awareness 44-45
2) Appointments & Resignations 46-47
3) Days & dates 48-49
4) Defence & Security 50
5) Honours and Awards 51-52
6) India and the World 53
7) Industry 54
8) Infrastructure 54
9) International Affairs 55
10) Polity and Governance 56
11) Reports and Indices 56-57
12) Sports 57-59
13) Miscellaneous 60-62

A STUDY IQ Initiative
bankiq.in May 2019

5 GROUP DISCUSSION Banking Terminology


TOPICS FOR BANK EXAMS
Bitcoin
o Jet Airways Crisis
o Should Public Sector Banks be Privatized Bitcoin(B) is a cryptocurrency, a form of electronic cash. It is a
o Plastic Money- Merits& Demerits decentralized digital currency without a central bank or single
o NYAY- Can it eliminate Poverty administrator that can be sent from user to user on the peer-to-peer
o Reservation in Education & Jobs- Is Justified? bitcoin network without the need of intermediaries.

TIP OF THE MONTH Inflation

Interview Do’s and Don’ts Inflation is a quantitative measure of the rate at which the
average price level of a basket of selected goods and services in
DO’S- an economy increases over a period of time. It is the constant
rise in the general level of prices where a unit of currency buys
Dress appropriately. Extremes in fashion or very casual clothes less than it did in prior periods. Often expressed as a
should generally be avoided. Look neat and clean. percentage, inflation indicates a decrease in the purchasing
An interview is a massive event and can change your life - so take it power of a nation’s currency.
seriously.
Think of your interview as a performance. Money Laundering
Most of the time you have 10 to 30 minutes to deliver your very best.
Get out of your comfort zone it is the only way you can grow if you Money laundering is the process of making large amounts of
hate interviewing, practice, practice, practice, practice this is the only way. money generated by a criminal activity, such as drug trafficking
Express yourself and your views clearly. or terrorist funding, appear to have come from a legitimate source.
Listen carefully to the questions and answer clearly and thoughtfully. The money from the criminal activity is considered dirty, and the
Make eye-contact. Remember to talk to the person (not the top right process "launders" it to make it look clean. Money laundering is
hand corner of the room or at their shoes). itself a crime.
Do your research on the bank-Do a thorough background research
of the bank and the job profile you're interviewing for. Visit the bank CAMELS Rating System
website before appearing for the interview.
Use quotes, facts and figures, statements, everyday life examples to CAMELS is a recognized international rating system that bank
express a clear chain of thoughts. Also it might leave a good impression supervisory authorities use in order to rate financial institutions
on the examiner and help you score well. according to six factors represented by its acronym. Supervisory
Thank the interviewer-Last but not the least; do not leave the room authorities assign each bank a score on a scale. A rating of one
without thanking the interviewer(s) for giving you his/her/their time. is considered the best, and a rating of five is considered the
Stay genuine. worst for each factor.

DON’TS- Masala Bonds

Don’t give any irrelevant information or excessive details. Masala bonds are bonds issued outside India but denominated
Never give an impression that you are not interested in a in Indian Rupees, rather than the local currency. Masala is an
particular bank. Indian word and it means spices. The issuance of rupee
You do not need to jump to answers. It’s OK to take a second or two denominated bonds transfers risk associated with currency
to compose your thoughts. Don’t rush. fluctuations to investors and not to the issuers. The term was
Do not defend yourself in wrong answers and get into an argument. used by the International Finance Corporation (IFC) to evoke
If you argue, they are better than you to do it. the culture and cuisine of India.
Avoid long pauses while speaking.
Do not lie. UPI

Unified Payments Interface (UPI) is an instant real-time


payment system developed by National Payments Corporation
of India facilitating inter-bank transactions. The interface is
Page 1
STUDY IQ
bankiq.in May 2019

regulated by the Reserve Bank of India and works by instantly


transferring funds between two bank accounts on a mobile Notes
platform.

Bancassurance

Bancassurance is an arrangement between a bank and an


insurance company allowing the insurance company to sell
its products to the bank's client base. This partnership
arrangement can be profitable for both companies. Banks
earn additional revenue by selling insurance products, and
insurance companies expand their customer bases without
increasing their sales force or paying agent and broker commissions.

Banking Ombudsman

The Banking Ombudsman Scheme was implemented by the RBI to


redress the complaints of customers on certain types of banking
services provided by banks and to facilitate the settlement of those
complaints. The Scheme was introduced under the Banking
Regulation Act of 1949 by RBI with effect from 1995. Later it was
legally refined and modified through the introduction of regulations
under Banking Ombudsman Scheme 2006. The latest version was
made in 2017. The banking Ombudsman actually is a senior official
appointed by the RBI.

Credit Crunch

A credit crunch is an economic condition in which investment capital


is hard to secure. Banks and investors become wary of lending funds
to individuals and corporations, which drives up the price of debt
products for borrowers. Often an extension of a recession, a credit
crunch makes it nearly impossible for companies to borrow because
lenders are scared of bankruptcies or defaults, resulting in higher rates.

Currency Chest

Currency chests are branches of selected banks where bank notes


and rupee coins are stored on behalf of the Reserve Bank of India for
further distribution of these notes and coins across the country
through bank branches in their area of operation. The RBI has set up
over 4,075 currency chests all over the country.

ABBREVIATIONS
BCBS-Basel committee on Banking Supervision.
CASA- Current A/C Saving A/C
DNS- Domain Name System
ECS- Electronic Clearing Service
FINO- Financial Inclusion Network and Operational Limited
IBBI- Insolvency and Bankruptcy Board of India
IFSC- Indian Financial System Code
MFI- Micro Finance Institutions
NECS- National Electronic Clearing System
TDS- Tax Deducted at Source
Page 2
STUDY IQ
bankiq.in May 2019

TOPIC IN Interesting Facts

Bara Bhangal village in Himachal Pradesh, recorded 100

NEWS
percent voting in the Lok Sabha 2019.
In constituency Navsari,-the highest victory margin was
recorded in 2019 General elections.BJP leader CR Patil won the
Nvasari Lok Sabha seat .Winning Margin –689668 votes
The lowest victory margin recorded in 2019 General Elections
is in Machhlishahr, Uttar Pradesh constituency. Winning
Margin –181 votes
In Himachal Pradesh the BJP had the highest vote share with
69.1% in Himachal Pradesh.
The world’s highest polling station is at Tashiganag, Himachal
Pradesh, with the height of 15,256 feet. Till 2017 the highest
polling station is Ka at Himachal Pradesh.
AAP is the only party to field a transgender candidate. Aam
Aadmi Party Filed transgender candidate, Chirpi Bhawani from
the Prayagraj seat in Uttar Pradesh.
INC party fielded the maximum women candidates.
In UP and West Bengal state the maximum women
candidates won the elections.
Modi 2.0 era begins; 58 – member team took charge Tejaswi Surya from Bangalore, South, became the youngest
BJP candidate to win in Lok Sabha Elections 2019. The
Narendra Modi was on 30th May sworn in for his second term as youngest candidate of LokSabha Elections in 2019 who
the Prime Minister at a glittering ceremony at Rashtrapati Bhavan. wonthe election is Chandrani Murmu, the BJD candidate from
With the BJP-led NDA having swept the Loksabha elections, Keonjhar.
winning 352 of 542 seats and securing a huge mandate, there are Bihar state saw about 8.17 lakh voters opting for none of the
strong expectations that Modi 2.0 will deliver fast-track reforms on two Above (NOTA) option, the highest in the country.
crucial factors of productivity land and labour. Modi, and his Cabinet 4 independent candidates won in Lok Sabha elections 2019.
colleagues were administered the oath of office and secrecy by Sangrur, Punjab is the only seat that AAP won in Lok Sabha
President Ram Nath Kovind at the ceremony, attended by about elections 2019–MP Bhagwant Mann.
6,000 invitees, including foreign dignitaries, BIMSTEC leaders,
Chief Ministers of States, RBI Governor Shaktikanta Das, corporate Hits and Misses : 5 years of Modi Government
honchos, including Ratan Tata, Mukesh Ambani and Tata Sons
chief N Chandrashekhar, besides sports and film personalities. Here are the hits and misses of the NDA government –
As many as 58 Ministers, including Prime Minister Modi, were sworn on
30th May. There were 25 Cabinet Ministers, nine Ministers of State (with GDP Growth: With Niti Aayog panel ‘downgrading’ UPA-era
independent charge), and 24 more Ministers of State.Therearethree growth of revised 2016-18 data, Modi government’s GDP
women Cabinet Ministers — Smriti Irani, Harsimrat Kaur Badal and growth record 7.6% average of 5 years is comfortably ahead of
NirmalaSitharaman — in the Modi 2.0 team. In all, Team Modi has six UPA’s 6.7%. This century’s fastest growth year still belongs to
women Ministers — three Cabinet and three Ministers of State. UPA (8.5% IN 2010-11). India overtook France in 2018 to become
Modi 2.0’s Cabinet team comprised almost all the seniors — except the world’s 6th largest economy. It will take 5th position from
ArunJaitley and Sushma Swaraj who opted out — of his earlier the UK this year on way to a $3 trillion GDP in 2020.
innings. BJP President Amit Shah, a first-time Member of Parliament, On Job creation, Govt found itself on a back foot: Slow job
was inducted into the Council of Ministers. creation- The labour ministry stopped releasing its quarterly
About 22 Ministers from the earlier Modi-led Government have survey of employment. The govt claimed jobs were being
been dropped. Some of the key Ministers who did not make it are created but not captured in official estimates. On Jan 30, a
Suresh Prabhu, Maneka Gandhi, JP Nadda, Radha Mohan Singh and media report on NSSO’s periodic labour survey said that annual
Jayant Sinha. employment rate in 2017-2018 was at a 45 –year high of 61%.
Some of the new faces in the Council of Ministers are Arvind CMIE, which publishes a monthly employment report, estimated
Sawant, Prahlad Joshi, Arjun Munda, Ramesh Pokhariyal and S that at least a million jobs were lost in 2018, and unemployment
Jaishankar. rate rose from 5% in January to 7.7% in December.
From the Opposition parties, the Congress was represented at the 5 Ministries get stellar score from people, and Finance is
swearing-in ceremony by UPA Chairperson Sonia Gandhi, not among them : The top five ministries of the Modi
Congress President Rahul Gandhi and former Prime Minister government are Defence, External Affairs, New and Renewable
Manmohan Singh.
Page 3
STUDY IQ
bankiq.in May 2019

Energy, Coal, and Road, Transport and Highways, a survey conducted weakness, Axis Capital attributes it to stress in rural income,

Industry
by LocalCircles showed. Indians have given a stellar score to the rising unemployment and insufficient job creation. In the long
Defence Ministry — 4.9 on the scale of 5. The government has been term, the government will need to focus on income growth to
applauded for “taking some bold steps like the surgical strikes against drive up household consumption.
Pakistan. Healthcare - BJP’s election manifesto promised setting up
Four ambitious targets NDA is poised to miss: After taking over of 1,50,000 health and wellness centres (HWCs) by 2022. It
the Prime Minister’s Office in 2014, Narendra Modi shared his vision to proposed a separate pricing policy for medical devices. BJP
make India an investor-friendly destination, which would subsequently has also promised to increase spending on healthcare to 2.5
lead to job creation and development. However, despite ambitious per cent of GDP by 2025 from 1.15 per cent now.
plans like Make in India, Narendra Modi, five years later, is poised to
miss some targets. What the Modi Government 2.0 has Promised?
4 developments that shocked the nation: From vowing to build
toilets in every school to announcing demonetisation to Moody’s
India has the world’s second largest startup ecosystem, and
surprise rating upgrade to massive fraud at Punjab National Bank
the BJP is looking to increase its support to the community
(PNB) allegedly by diamond czar Nirav Modi, the nation has been
by easing regulatory requirements for them. The party said in
shocked in a good and a bad way in last five years.
its manifesto that over 17 crore entrepreneurs have availed
Has Modi kept India secure? Are women now safer in India? :
loans under the PradhanMantri Mudra Yojana, and expects
The BJP government says it brought in tough laws to combat violence
to take the total number of beneficiaries of loans to go up to 30
against women. The data shows a rise in the reporting of rape to the
crore people. The ruling party also said it was aiming to create
authorities, particularly after the notorious Delhi gang rape of 2012.
50,000 new startups by 2024, and establish 100 innovation
But the conviction rate for cases that go to the courts has not really
zones in urban local bodies. In addition, the party said it
improved in the last few years
would set up about 500 new incubators and accelerators
by 2024.
Sectors crying for urgent attention in Modi 2.0 regime.-

As Prime Minister Narendra Modi begins his second innings, a few


Notes
ailing sectors of the economy are looking for actions from the new
government to get them out of the rut.
Aviation – The Jet Airways saga is the latest instance that reflects
the tough conditions that the domestic aviation industry is going
through. The Naresh Goyal-led airline in April shut down operations,
going the Kingfisher’s way, rendering nearly18,000 employees jobless.
Experts expect the government to bring aviation turbine fuel (ATF)
under the GST ambit, which will remove tax disparity across states. At
present, ATF in India is priced 30-40 per cent above global rates
owning to higher taxation. There are also demands that the
government should facilitate setting up of maintenance & overhaul
facilities (MRO) within India, which can help cut costs.
Power- Mounting losses and piling debt at the Discoms have been
a worry for the power sector. The Modi government in its first term
formulated UDAY in a bid to improve the financial health and
operational efficiency of the country’s debt-ridden power distribution
companies. Its aim was to bring registered losses of Discoms to below
15 per cent by FY19 and reduce the gap between average cost of
supply and average revenue realised to zero. Data showed electricity
generation grew by a modest 3.6 per cent in FY19.
Banking NBFCs – The rising stress in the NBFC sector has kicked off
a debate over whether it is an insolvency issue or just a liquidity
crunch. Providing state-run banks with adequate capital is another
issue the government needs to address quickly. The brokerage
estimated that the government would need $30-35 billion to restart
growth in state-owned banks, with roughly half of it just to meet
regulatory capitalisation needs.
Consumption - Sectors across the consumption theme – both
staples and discretionary – have been seeing a slowdown. While credit
squeeze and diminishing wealth effect are the primary reasons of the
Page 4
STUDY IQ
bankiq.in May 2019

Here’s the full list of the Union Council of Ministers and their portfolios, under the Modi Government 2.0.-

Union Minister Constituency Portfolio

Narendra Modi Varanasi Prime Minister, Ministry of Personnel, Public


Grievances and Pensions; Depar tment of
Atomic Energy; Department of Space; All
important policy issues; and All other
portfolios not allocated to any Minister .
Rajnath Singh Lucknow Minister of Defence
Amit Shah Gandhinagar Minister of Home Affairs
NitinGadkari Nagpur Minister of Road Transport and Highways;
and Minister of Micro, Small and Medium
Enterprises
D.V. Sadananda Bangalore North Minister of Chemical and Fertilizers
NirmalaSitharaman Karnataka Rajya Minister of Finance; and Minister of
Sabha Corporate Affairs
Ramv ilasPaswan - Minister of Consumer Affairs, Food and
Public Distribution
Narendra Singh Tomar Morena Minister of Agriculture and Farmers Welfare;
Minister of Rural Development; and Minister
of Panchayati Raj
Patna Sahib Minister of Law and Justice; Minister of
Ravi Shankar Prasad Communications; and Minister of Electronics
and Information Technology
HarsimratKaurBadal Bathinda Minister of Food Processing Industries
Thaawar Chand Gehlot Madhya Pradesh RS Minister of Social Justice and Empowerment
Dr . S Jaishankar Minister of External Affairs
Ramesh Pokhriyal Nishank Haridwar Minister of Human Resource Development
ArjunMunda Khunti Minister of Tribal Affairs
SmritiIrani Amethi Minister of Women and Child Development;
Minister of Textiles
Dr. H arsh Vardhan Chandni Chowk Minister of Health and Family Welfare;’
Minister of Science and Technology; and
Minister of Earth Sciences
PrakashJavadekar Maharashtra Minister of Environment, Forest and Climate
Change; and Minister of Information and
Broadc asting
PiyushGoyal Maharashtra Minister of Railways; and Minister of
Commerce and Industry
DharmendraPradhan Madhya Pradesh Minister of Petroleum and Natural Gas; and
Rajya Sabha Minister of Steel
Mukhtar Abbas Naqvi Jharkhand RS Minister of Minority Affairs
Pralhad Joshi Dharwad Minister of Parliamentary Affairs; Minister of
Coal; and Minister of Mines
Dr.MahendraNathPandey Chandauli Minister of Skill Development and
Entrepreneurship
ArvindSawant Mumbai -South Minister of Heavy Industries and Publ ic
Enterprise
Giriraj Singh Begusarai Minister of Animal Husbandry, Dairying and
Fisheries
Gajendra Singh Jodhpur Minister of Jal Shakti
Shekhawat

Page 5
STUDY IQ
bankiq.in May 2019

Ministers of State (Independent Charge) -

Santosh Kumar Gangwar Minister of State (Independent Char ge) of the Ministry of Labour
and Employment.
RaoInderjit Singh Minister of State (Independent Charge) of the Ministry of Statistics
and Programme Implementation; and Minister of State
(Independent Charge) of the Ministry of Planning
ShripadYessoNaik Minister of State (Independent Charge) of the Ministry of Ayurveda,
Yoga and Naturopathy, Unani, Siddha and Homoeopathy (AYUSH);
and Minister of State in the Ministry of Defence.
Dr.Jitendra Singh Minister of State (Independent Charge) of the Ministry of
Dev elopment of North Eastern Region;
Minister of State in the Prime Minister’s Office; Minister of State in
the Ministry of Personnel, Public Grievances and Pensions; Minister
of State in the Department of Atomic Energy; and
Minister of State in the Departmen t of Space.
KirenRijiju Minister of State (Independent Charge) of the Ministry of Youth
Affairs and Sports; and Minister of State in the Ministry of Minority
Affairs.
Prahalad Singh Patel Minister of State (Independent Charge) of the Ministry of Culture;
and Minister of State (Independent Charge) of the Ministry of
Tourism.
Raj Kumar Singh Minister of State (Independent Charge) of the Ministry of Power;
Minister of State (Independent Charge) of the Ministry of New and
Renewable Energy; and Minister of S tate in the Ministry of Skill
Development and Entrepreneurship
Hardeep Singh Puri Minister of State (Independent Charge) of the Ministry of Housing
and Urban Affairs; Minister of State (Independent Charge) of the
Ministry of Civil Aviation; and Minister o f State in the Ministry of
Commerce and Industry.
Mansukh L. Mandaviya Minister of State (Independent Charge) of the Ministry of Shipping;
and Minister of State in the Ministry of Chemicals and Fertilizers.

STUDY IQ Page 6
bankiq.in May2019

Ministers of State -

FaggansinghKulaste Minister of State in the Ministry of Steel.


Ashwini Kumar Choubey Minister of State in the Ministry of Health and Family Welfare.
Arjun Ram Meghwal Minister of State in the Ministry of Parliamentary Affairs; and
Minister of State in the Ministry of Heavy In dustries and Public
Enterprises.
General (Retd.) V. K. Minister of State in the Ministry of Road Transport and Highways.
Singh
Krishan Pal Minister of State in the Ministry of Social Justice and
Empowerment.
DanveRaosahebDadarao Minister of State in the Ministry of Consumer Affairs, Food and
Public Distribution.
G. Kishan Reddy Minister of State in the Ministry of Home Affairs.
ParshottamRupala Minister of State in the Ministry of Agriculture and Farmers Welfare.
RamdasAthawale Minister of State in th e Ministry of Social Justice and
Empowerment.
SadhviNiranjanJyoti Minister of State in the Ministry of Social Justice and
Empowerment.
Babul Supriyo Minister of State in the Ministry of Environment, Forest and Climate
Change.
Sanjeev Kumar Balyan Minist er of State in the Ministry of Animal Husbandry, Dairying and
Fisheries.
Dhotre Sanjay Shamrao Minister of State in the Ministry of Human Resource Development;
Minister of State in the Ministry of Communications; and Minister of
State in the Ministry of E lectronics and Information Technology.
Anurag Singh Thakur Minister of State in the Ministry of Finance; and Minister of State in
the Ministry of Corporate Affairs.
Angadi Suresh Minister of State in the Ministry of Railways.
Channabasappa
NityanandRai Minister of State in the Ministry of Home Affairs.
Rattan LalKataria Minister of State in the Ministry of Jal Shakti; and Minister of State
in the Ministry of Social Justice and Empowerment.
V. Muraleedharan Minister of State in the Ministry of External Affairs; and Minister of
State in the Ministry of Parliamentary Affairs.
Renuka Singh Saruta Minister of State in the Ministry of Tribal Affairs.
SomParkash Minister of State in the Ministry of Commerce and Industry.
RameswarTeli Minister of State in th e Ministry of Food Processing Industries.
Pratap Chandra Sarangi Minister of State in the Ministry of Micro, Small and Medium
Enterprises; and Minister of State in the Ministry of Animal
Husbandry, Dairying and Fisheries.
KailashChoudhary Minister of Sta te in the Ministry of Agriculture and Farmers Welfare.
DebasreeChaudhuri
Minister of State in the Ministry of Women and Child Development.

Page 7
STUDY IQ
bankiq.in May 2019

Banking and Madras HC sets aside its single-judge order


on NBFC registration

Financial In a victory to Reserve Bank of India (RBI), a division bench

Awareness
of the Madras High Court has set aside a single judge order
that ordered restoration of Certificate of Registration (CoR)
of non-banking financial companies (NBFC) cancelled by the
central bank for non-compliance of the enhanced net owned
fund (NOF) norms. .
RBI had notified an enhanced NOF holding by all NBFCs on
March 27,2015, which stipulated them to have Rs 1 crore as NOF
as of April 1, 2016 andRs 2 crore as of April 1, 2017.RBI had
cancelled the CoR of over 1,500 NBFCs for non-compliance.
The bench also directed the NBFCs to approach the appellate
authority within 30 days and raise their concerns to get the
grievances redressed.

RBI 25th Governor- Shaktikanta Das


Headquarters- Mumbai
Founded- 1april 1935, Kolkata.
o A Non Banking Financial Company (NBFC) isa company
registered under the Companies Act, 1956 of India,
engaged in the business of loans and advances, acquisition
of shares, stock, bonds, hire-purchase insurance business
SEBI finds NSE guilty but does not include any institution whose principal
The markets regulator on 30th April cracked the whip on National business includes agriculture, industrial activity or the sale,
Stock Exchange of India Ltd (NSE), and ordered to deposit nearly Rs. purchase or construction of immovable property.
1,000 crore in an investor fund and barring it from accessing capital
markets for six months, for lapses in its algorithmic trading systems Declare IL&FS as NPAs
and co-location services.
NSE had failed to ensure equal and fair access. National Company Law Appellate Tribunal (NCLAT) headed
SEBI also punished Ravi Narain and Chitra Ramakrishna, two by Chairman Justice SJ Mukhopadhayaon 2nd May, allowed the
former CEOs and founding members and asked to deposit 25% of banks to declare as non-performing assets the accounts of
their salary. IL&FS and its group companiesthat defaulted on payments.
The appellate tribunal has also clarified that although the
banks would declare the IL&FS accounts as NPAs but cannot
initiate recovery process and debit money.

NPAs- Non-Performing Assets


IL&FS- Infrastructure leasing and Financial Services

Finance ministry confirms slowdown fears

The GDP growth may have touched 6.5%in the March


SEBI-Securities and Exchange Board of India
quarter, against 6.6% in the previous quarter
Establishment of SEBI- April12, 1992
The slowdown in the economy in FY19 has been caused by
Headquarter- Mumbai, Maharashtra
declining growth of private consumption, tepid increase
Chairperson-Ajay Tyagi
in fixed investment and muted exports.
Current account deficit (CAD) situation may have improved
NSE Founded in- 1992
in the last quarter of FY19 from 2.5% in the December quarter,
MD & CEO- VikramLimaye
as dip in imports has improved the merchandise trade deficit.
Headquarter- Mumbai, Maharashtra
The Central Statistics Organisation has pegged FY19 GDP
growth at 7%, down from 7.2% in FY18 and 8.2% in the year
o The NSE is the second largest in the world by nos. of trades
before.
and equity shares from January to June 2018, according to the
World Federation of Exchanges (WFE) report.
Page 8
STUDY IQ
bankiq.in May 2019

Though the easing of the monetary policy (the repo rate has been RBI imposes penalties on five PPI issuers
cut by 25 basis points each in February and April) has the potential
to support growth. Credit growth could have been challenged by Reserve Bank of India (RBI) has imposed penalties on five
continuous tightening of bank liquidity, causing the call money prepaid payment instrument (PPI) issuers for violating
market rates to trend up since Q1. regulatory norms under Section 30 of provisions of Payment
and Settlement System Act, 2007.
o Gross domestic product (GDP) is a monetary measure of the It includes Vodafone’s m-pesa, My Mobile Payments Limited
market value of all the final goods and services produced in a period which own Oxigen, Flipkarts’sPhonePe, Y-cash Software
of time, often annually. Solutions Private Limited which owns YPayCash and GI
o Current finance minister- PiyushGoyal Technology Private Limited.

RBI 25th Governor- Shaktikanta Das


IBC Resolution Headquarters- Mumbai, Maharashtra
Founded- 1april 1935, Kolkata
Over two dozen companies are named by the Reserve Bank of
India in its 2nd list of large non-performing assets (NPAs), but only
three companies (Ruchi Soya Industries, EPC, Constructions and
SBI links its interest rate to Repo rate
ARGL) have come close to being resolved successfully through the
State Bank of India (SBI), linked its interest rates on savings
corporate insolvency resolution process (CIRP) since August 2017.
bank deposits and short term loans to repo rate of Reserve
There successful bidders arePatanjaliAyurvedfor Ruchi Soya,
Bank of India (RBI). The repo rate is interest rate at which
Royale Partners for EPC Constructions and CarVal-Arcilfor ARGL.
RBI lends short term funds to banks. At present the repo
The second list involved companies with a total principal
rate is 6%.
outstanding of Rs 1.28 lakh crore,March 2018 report by CLSA.
SBI has linked savings bank deposits to repo rate with
RBI 25th Governor- Shaktikanta Das balances of more than Rs 1 lakh. The advantage of linking to
Headquarters- Mumbai, Maharashtra repo rate is to expect to bring more transparency in fixing
Founded- 1april 1935, Kolkata. interest rates and help in faster transmission of rates.

SBI Chairperson-Rajnish Kumar


Recommendation of FPI investment in Headquarters- Mumbai
unlisted companies Founded in- 1 July 1995

Jalan Panela

Jalan panel was appointed on December 26, 2018, with six


members to look into the appropriate capital reserves the
RBI should maintain. The committee was supposed to
submit its report in 90 days from the first day of its meeting.
The panel has been entrusted with the task of reviewing the
best practices followed by central banks worldwide in making
assessment and provisions for risks.
The panel has sought six months extension.

o The Reserve Bank of India, in consultation with the


government, constituted acommittee led by the central
bank's former Governor BimalJalan to look into its Economic
The Securities and Exchange Board of India (SEBI) committee is Capital Framework.
set to recommend that foreign portfolio investors be allowed to
buy shares of unlisted companies.
Such a measure would be a boost for start-ups and other unlisted
State Finance Commissions
entities as they will get access to a broader pool of capital.
There is a Meeting held between RBI Governor Shaktikanta
o The Securities and Exchange Board of India (SEBI) is the regulator Das and Finance Commission Minister N.K Singh, to setting
for the securities market in India. It was established in 1988 and given up ‘State Finance Commissions’ for state governments and
statutory powers on 30 January 1992 through the SEBI Act, 1992. also public sector borrowing requirements and continuity
o Headquarter- Mumbai, Maharashtra of the Finance Commission were also discussed.

Page 9
STUDY IQ
bankiq.in May 2019

“It was felt that this was required more in view of the fiscal Finance Commissionand Finance Ministry discussions
management requirementsof the states.Importance of the states
in the economy has increased with the shift in composition of Finance Commission held discussions with the Finance
government finances.” RBI Ministry,related with-
“Specific factors drive fiscal slippages: these factors include UDAY A. Fiscal management
in the past and farm loan waivers and income support schemes in B. Economic management
2018-19 RE”. C. Rationalisation of expenditure related to centrally
sponsored schemes.
Canara Bank and Canara HSBC OBC launch 15th Finance Commission observed that the GDP numbers
‘ Webassurance’ suggest continued high growth over the medium term eventhough
there have been fluctuations within the overall global trend.
Canara Bank and its life insurance partner Canara HSBC Oriental The Commission also noted that the revenue projections on
Bank of Commerce Life Insurance launched ’Webassurance’ to direct taxes are healthy though on Indirect taxes there have
enable its customers to purchase life insurance in a convenient and been periodic fluctuations, chaired by N K Singh.
hassle-free way through bank’s website.
Canara HSBC OBC Life is jointly owned by two public sector banks, RBI’s vision document
Canara Bank (51 per cent) and Oriental Bank of Commerce (23 per
cent) and HSBC Insurance (Asia Pacific) Holdings (26 per cent), the The RBI’s ‘Payment Systems Vision 2021’ document would
Asian insurance arm of banking and financial services groups HSBC. act as a catalyst for promoting digital economy and instill
confidence among the general public. Aiming at a‘cash-lite’
Canara Bank Chairman-Shri T.N. Manoharan
society, the Reserve Bank of India released the vision document
Headquarters- Bangalore
for ensuring a safe, secure, convenient, quick and affordable
MD & CEO- R A Sankara Narayanan
e-payment system as it expects the number of digital
transactions to increase more than four times to 8,707 crore in
Retail Inflation December 2021.
“We are confident with our vision as a payment system
Retail inflation inched up to a six-month high of 2.92 per cent in provider aligned with the regulators, we will be able to
April due to a spike in food prices, including vegetables, meat, fish contribute in increasing the digital transactions penetrations
and eggs. especially in the assisted segment in smaller cities and rural
Inflation based on the Consumer Price Index (CPI) was at 2.86 per India,”- RBI
cent in the previous month and 4.58 per cent in April 2018. The
rate of price rise in April is the highest since October 2018 when the PNB merger
rate was 3.38 per cent.
The Reserve Bank mainly factors in the CPI-based inflation while
setting the key policy rates. The Monetary Policy Committee headed
by the RBI governor would be meeting early June to decide on its
monetary policy.

Punjab National Bank is looking to merge with two or three


government owned banks like Oriental Bank of Commerce,
Andhra Bank and Allahabad Bank.
Government is trying to cut the number of state-owned
PSBs set aside Rs 52,739 cr. as provision in Q4FY19 lenders. This could be 1 of the first decisions taken by the
new government.
Stung by the collapse of IL&FS and Jet Airways Banks struggle under a huge debt pile: 9 trillion Indian
13 Public Sector Banks have put aside Rs52,739 crore in the March Rupees ($130 billion), or nearly 5% of the nation’s gross
quarter for existing and potential loan losses. domestic output.To revive the banking sector the
government of India has injected about $36 billion of tax
payer money in the last five years and also initiated an asset
quality review of banks by the central bank.

Page 10
STUDY IQ
bankiq.in May 2019

PNB founded in- 1894 Finance Ministry will soon appoint an advisor to explore
Headquarter - New Delhi its feasibility.
Chairman and MD- Sunil Mehta The balance sheet of PSU banksexpected to strengthen
going forward and an ETF would provide risk-averse investors
Oriental Bank of Commerce founded in- 1943 an option to hold sharesof multiple banks through a single
Headquarter – Gurugram, India financial instrument.
MD & CEO – Mukesh Kumar Jain
Provide names of big loan defaulters : CIC to RBI
Andhra Bank founded in- 1923
Headquarter- Hyderabad The Central Information Commission (CIC) has directed the
MD & CEO – J. Packirisamy Reserve Bank of India to disclose the list of big loan
defaulters it has sent to banks for resolution.
An Economic agenda for the next government In 2017 RBI Deputy Governor Viral Acharya said that the
accounts of some loan defaulters have been sent to banks
“A wish list for any new government will be to have political will in for resolution.
the first year in office to bring in reforms in Land, Labour and The RBI directed banks to file insolvency applicationsagainst
Capital”- N.K Singh, 15th Finance Commission. 12 large accounts comprising about 25% of the total NPAs.
“Some important structural reforms like the GSTandIBC are done, Nutan Thakur, a social activist wanted to know from the RBI the
although some more tweaking is required there. These have to be list of loan defaulters, The RBI had refused to provide the details.
followed up with reforms in the factors markets”- Krishnamurthy
Subramanian, Chief economic advisor. MCA lens on IL& FS shareholders
“Fixing the NBFC crisis and growth slowdown are immediate
priorities. Mobilise more revenues and channel them towards The probe into the collapse of Infrastructure Leasing &
productive spending”- SonalVarma,Chief India economist at Nomura. Financial Services is set to expand to include the role of
“First, the government has to address growth slowdown, come out marquee shareholders such as HDFC, State Bank of India
with policies to fix farm distress and more reforms, especially in GST and Orix Corporation.
and IBC fast. It needs to seriously look at more divestment revenues”- How these shareholders are their representatives on the
DK Joshi, Chief economist at Crisil. IL&FS board failed to flag the irregularities and deteriorating
“The economy is facing risks of getting into a middle-income trap. financial condition.
Schemes like PM-Kisan are sort of compensatory apology; they may An investigation will also be conducted into the role of CARE,
stimulate consumption in a limited manner but won’t be enough to India Ratings and ICRA which had given high ratings to IL&FS
reverse structural slowdown”- Rathin Roy, Director, NIPFP. before the crisis.
“The slowdown in private consumption is here to stay for some time
and no government can stimulate it overnight. So economic growth
will stagnate around 7% for one-two years before recoding a gentle
recovery.”- MadanSabnavis, Chief economist at CARE Ratings.
“Economic growth has been weakening in the past four-six
quarters, which needs to be corrected urgently through appropriate
policy reforms”- ArvindVirmani, Former chief economic advisor.

NPA crisis
Below 7% Economic growth- ET Poll
With the economy seeing a very sharp slowdown with demand
weakening and some uncertainty on the monsoon, exposure to Because of a disappointing 4th quarter India’s economic
businesses in sectors such as MSME, real estate, agriculture, power, growth is likely to have slipped below 7% in FY19.
and even to NBFCs could go bad. The economy is facing tighter financial conditions, weak
The contraction in factory output in March and also the very poor global and domestic demand, with private consumption
corporate results for Q4FY19. For the MSME sector, interest costs have slowing materially
not fallen meaningfully as banks have not transmitted the entire
reduction in the repo rate to customers. Did you Know ?
Finance Ministry to launch ETF with banks stocks, Did you know when the symbol for Rupee was adopted?
financial institutions this fiscal
In 2010 Indian rupee symbol Rupee “₹” was adopted. D.Udaya
After the resounding success of CPSE ETF (exchange traded Kumar is the creator of the rupee symbol. The symbol is a
fund) and Bharat-22 ETF, the government is looking at combination of the Latin Letter “R” and Devanagari letter “₹”.
diversifying the ETF basket by including stocks public sector
banks and financial institutions. Page 11
STUDY IQ
bankiq.in May 2019

But the reverse happened. As many as eight of the 13

Taxation
product segments witnessed a rise in purchases from
overseas since the tariffs went up.
Between October 2018 and February 2019, imports on
air conditioners and refrigerator to footwear rose an
average 5% to $2,343.41 million.
Gems and Jewellery segment, already down by 10.8%
up to September last fiscal.

No drop in Income Tax return

The Central Board of Direct Taxes has said figures for income
tax returns e-filed during the financial year 2018-19 as
GST collection jumps up compared to 2017-18 are not comparable. During FY 2017-18,
out of a total of 6.74 crore ITRs which were e-filed, 5.47 crore
The goods and services tax (GST) collections for the first month of ITRs were filed for Assessment Year (AY) 2017-18, the current year.
the financial year 2019-20 came in at Rs 1.14 lakh crore, highest level In comparison, during FY 2018-19 only 0.14 crore ITRs for AY
for any month since July 2017. 2017-18 were filed during FY 2018-19 as these were the revised
Though April collections are typically high since business tends to ITRs for the relevant AY which could only be filed due to change
close the year’s accounts. GST collections are being taken onto a in law
higher plane, finally.
The collections have risen to record levels, collection in March were Anti- profiteering mechanism
Rs1.07 lakh crore, the highest level until then.
In April 2019 collections were 10% higher than the level in the
year-ago month and a solid 16% higher than the monthly average
in 2018-19.
The Centre’s GST revenue or FY20 is projected at Rs 7.61 lakh crore.

Different high courts in the country have given stay orders


on several fiats of the National Anti-profiteering Authority
(NAA) for the goods and services tax (GST).
Casting doubts on the legal tenability of the way the
nearly one-and-half-a-year-old set-up operates and passes
orders on alleged cases of profiteering by businesses.
5 firms have got reliefs from the high courts.
Court has questioned: set of rules or methodology to
determine what constitutes ‘profiteering’ and how to
calculate the amounts garnered by a business through it.
o Goods and Services Tax (GST) is an indirect tax (or consumption
NAA was constituted in November 2017 for period of two
tax) imposed in India on the supply of goods and services. The tax
years. it has delivered 60 orders so far where it found
rates, rules and regulations are governed by the GST Council which
businesses guilty of profiteering in over a dozen cases.
consists of the finance ministers of centre and all the states.

September 2018 Duty Hike & its effect

In September 2018 the government raised basic customs duties


on scores of products, to crack down on ”non-essential imports” and
contain their debilitating impact on current account deficit (CAD).

Page 12
STUDY IQ
bankiq.in May 2019

Appellate Authority for Advance Ruling (AAAR)


Notes
The GST Council is likely to consider a proposal for setting up a
national bench of the Appellate Authority for Advance Ruling
(AAAR) to reconcile the contradictory orders on similar issues
passed by AAARs in different states, a move aimed at providing
certainty to taxpayers.
An aggrieved party can file an appeal against an order of the AAR
to the AAAR within a period of 30 days, which may be further
extended by a month.

Single authority for sanctioning, processing GST refunds

Finance Ministry is looking to speed up and simplify the process


for exporters.
The current mechanism entails a twin refunds sanctioning
authority of the central and state tax officers.
As per the system being worked out by the Revenue Department,
the taxpayer will get full refund from his jurisdictional officer once
the claim is sanctioned.

GST Evaders Can Be Arrested

Telangana High Court judgement held that a person can be


arrested by the competent authority in cases of Goods and Service
Tax (GST) evasion.
The Supreme Court has dismissed a plea challenging Telangana
High Court judgement.
Sub Section (1) of Section 69 of the Act empowers the
Commissioner to order the arrest of a person, when such a person,
when such a person is believed to have committed a cognizable and
non bailable offence.

Page 13
STUDY IQ
bankiq.in May 2019

Agriculture
Notes

Farmer producer companies (FPCs) of Maharashtra

Farmer producer companies (FPCs) of Maharashtra have turned out


to be a source of succour for farmers badly affected by drought in the
state.
These farmer producer companies claim to have provided
employment to at least 11,900 farmers at a time of financial distress.
MAHA-FPC has entered into a joint venture with the National
Agricultural Cooperative Marketing Federation (NAFED) to develop a
value chain for onion procurement in Maharashtra.
The Rs 25-crore project will see the government making 50%
investment while the rest will be raised by NAFED and FPCs.

Page 14
STUDY IQ
bankiq.in May 2019

Industry

India’s oil import dependence jumps to 84%

Prime Minister NarendraModi may have set a target to cut India’s


oil import dependence by 10 per cent. On March 2015, He had said Poor Corporate profits
that India needs to bring down its oil import dependence from 77 per
cent in 2013-14 to 67 per cent by 2022 when India will celebrate its Q4FY19 have been quiet poor. 1,012 companies (excluding
75th year of Independence. banks and financials), net profits fell 13% year-on-year in the
Itchanged exploration rules multiple times. The previous New three months to March.
Exploration Licensing Policy (NELP) was changed to Hydrocarbon Demand is very weak and with purchasing power crimped,
Exploration and Licensing Policy (HELP) that gave companies the competitive intensity has gone up.
freedom to choose area they want to explore. EBITDA excludes interest, depreciation, amortization and
India spent USD 111.9 billion on oil imports in 2018-19, up from taxes, EBITDA margin can provide an investor business owner
USD 87.8 billion in the previous fiscal year. or financial professional with a clear view of a company’s
operating profitability and cash flow.EBITDAmargin is an
Industrial output assessment of a firm’s operating profitability as a percentage
of its total revenue. It is equal to earnings before interest, tax,
India’s industrial output declined by 0.1% in March because of depreciation and amortization (EBITDA) divided by total
manufacturing, capital goods and consumer durables. revenue.
Manufacturing , which constitutes 77.63% of the Index of Industrial
Production (IIP)
The previous low for IIP was a 0.3% decline in June 2017.

CCI ON Maruti

India’s antitrust regulator is looking into allegations that Maruti


Suzuki, resorted to anti-competitive practices by controlling its
dealer discounted cars.
Maruti forces its dealers to limit the discounts they offer.
It is up to the dealer to offer discounts and take a lower profit
margin, which they often do.
In 2017, South Korean firm Hyundai Motor Co’s India unit was fined
$12.5 million by CCI for antitrust violations including resale price
maintenance.

Page 15
STUDY IQ
bankiq.in May 2019

Infrastructure
Notes

ADB to fund Indian infra projects

Asian Development Bank (ADB) has given in principle nod for


financing four metro rail projects and Rs 30,000 crore rapid rail
corridor between Delhi and Meerutwith the Centre, as part of its
effort to improve urban transport system in India. Other Projects
especially expansion of Chennai and Bengaluru Metro are also under
consideration.
ADB has funded Jaipur Metro and expansion of Mumbai Metro in
the past.

Formation of ADB- 1966


Headquarters- Manila, Philippines
President- TakehikoNakao

Page 16
STUDY IQ
bankiq.in May 2019

The Trump administration hit Chinese telecoms giant

International
HUAWEI with severe sanctions and said it was adding HUAWEI
Technologies Co Ltd and 70 affiliates to its “Entity List”.
HUAWEI denies its products pose a security threat, said it
was “ready and willing to engage with the U.S. government and

Affairs come up with effective measures to ensure product security”.


The United States has temporarily eased trade restrictions on
China’s HUAWEI to minimize disruption for its customers. US
alleges that the firm is involved I activities contrary to
national security.
The world’s largest telecoms equipment maker said meant
little because it was already prepared for U.S. action.
The two countries increased import tariffs on each other’s
goods over the past two weeks.

Xi Jinping seeks global cooperation on technology

China International Big Data Industry Expoheld in Guiyang


Countries should cooperate in developing the Internet, big
data and artificial intelligence.
China attaches great importance to the development of the
big data industry and is willing to explore growth path with
USA China Trade War other nations for new technologies.
The US government has announced a ban on exports from
Huawei and barred the Chinese company from buying
components from American firms.
Shares of semiconductor and chip suppliers have plunged
over the past month amid trade-war tensions.

Notes

As per a notice posted to the Federal Register, USA would raise


tariffs on $200 billion worth of Chinese imports from current 10% to
25%. The reason behind this is that, China had backtracked on almost
all aspects of a draft trade agreement with the United States.
China said that it is forced to retaliate.
Moody’s: an all-out trade conflagration between the world’s
two-largest economies risked tipping the U.S. economy into recession
by the end of 2020 just as voters go to the polls in the U.S.

China vs USA

China would raise tariffs on $60 billion in U.S. goods in retaliation


for the U.S. decision to hike duties on Chinese goods. Beijing will
increase tariffs on more than 5,000 products to as high as 25%.
Duties on some other goods will increase to 20%.

U.S. blacklists China’s HUAWEI


Page 17
STUDY IQ
bankiq.in May2019

India and
the World
H- 1B fee hike

Trump administration could soon increase the H- 1B visa fee


further, pushing IT companies to hire locally in the US.
US Labour Secretary Alexander Acosta told US lawmakers
that an increase in H- 1B visa application fee will raise funding
for the expansion of an apprentice programme, which trains
American youths in technology-related activities.

India- US Trade

India-Pakistan trade halves

India’s imports from Pakistan crashed 44.6%and its exports


plunged 43.7% in February when New Delhi revoked its most
INDIA-“If India scrap or trim the duties, only China and Hong
favoured nation (MFN) status of Islamabad in the wake of the
Kong will be the biggest beneficiaries of the move, USA will
Pulwama terror attack on the 14th of the month, perpetrated by
hardly gain.”
Pakistan-based militant outfit Jaishe-Mohammed (JeM)
US made up for only 2% (or $415 million) of India’s imports
Analysts said trade with Pakistan might have collapsed even
of these seven products worth $20.5 billion in financial year
further in March.
(FY) 18. India will lose as much as $3.2 billion a year in
India has slapped a 200% duty on purchases from Pakistan after
customs revenue.
the withdrawal of the MFN status the massive fall in exports suggests
India’s potential revenue loss will be way above the export
Islamabad has quietly raised its non-tariff barriers for Indian
incentives of $190 million that the US offered India in FY18
products in response to New Delhi’s tariff war.
under the so-called generalised system of preferences (GSP).
The US has indicated that it could hold its proposed
US cannot ensure crude sale to India at withdrawal of incentives on annual Indian Exports of $5.6
concessional rate billion under the so-called generalised system of preference
(GSP) until the formation of the new government here.
The US, said that it cannot ensure the sale of its crude oil to
India at concessional rates to make up for the cheaper Iranian oil
going out of the market.”
India stopped importing crude oil from Iran following the US move
to end sanction waivers. Iranian oil was a lucrative buy for Indian
refiners as the Persian Gulf provides 60 days of credit for purchase.
Iran also providing insurance cover during transit.
US Ambassador to India Kenneth Juster said, “The US working with A group of 25 influential American lawmakers has urged the
other countries, including Saudi Arabia, to ensure an adequate US Trade Representative not to terminate the GSP programme
supply of oil”. with India after the expiry of the 60-day notice on May 3,
India was the second-biggest buyer of Iranian crude oil after China. saying the country's companies seeking to expand their exports
to India could be affected.

Page 18
STUDY IQ
bankiq.in May 2019

The Generalized System of Preference (GSP) is the largest and


oldest US trade preference programme and is designed to promote India is not a Tariff King
economic development by allowing duty-free entry for thousands of
products from designated beneficiary countries. US President Donald Trump has labelled India the “tariff king”.
On 31st May, The Trump administration has said it is not going According to the WTO data, New Delhi has exercised
back on its decision to terminate India’s designation as a “maximum self-restraint” in taxing imports.
beneficiary developing nation. Its decision to end preferential tariffs India’s average bound rate (Maximum rate of tariff allowed
to $5.6 billion of Indian exports under GSP from June 5. - by World Trade Organisation (WTO) to any member state for
imports from another member state) is as much as 48.5%.
India’s actual applied tariff is as low as 13.8%.
Wilbur Ross on India: ICT products (20%), motorcycles
(50%), automobiles (60%) and alcoholic beverages (150%).
US charges 350% on tobacco, 163.8% on peanuts, 48% on
footwear, 38% on glassware for toilet and 32% on shoes.

India-China Trade

India has sought greater access for agricultural and animal


husbandry products in Chinese market to boost our exports and to
Bridge trade deficit. WTO’s dispute body
Necessary documents for bovine meat, milk and milk products have
been submitted to China’s General Administration of Customs of As many as 17 developing nations, including India and China
China (GACC). pitched for resolving the deadlock over the appointment of
China does not permit imports of Indian meat due to fears over members to the WTO’s appellate body at the earliest.
Foot and mouth disease. India answered that it is exporting meat to These nations say that it could lead to a complete paralysis in
as many as 70 countries since several years. the dispute settlement mechanism of the multilateral trade
The ban was imposed on apprehensions of melamine presence in body by December if members don’t act fast. This was part of
some milk consignments from China. Melamine is a toxic chemical a declaration released after a two-day ministerial meeting of
used for making plastics and fertilisers. 22 developing and least developed member countries of the
India also wants a great market access for pharmaceuticals. World Trade Organisation (WTO) in New Delhi. The Delhi
Declaration was signed by 17 of the 22 participating countries.
The declaration also made a strong case for preserving
flexibilities that are being offered to developing countries
under the WTO’s special and differential treatment (S&DT)
provisions, as they are rights of developing nations.
S&DT is one of the main defining features of the
multilateral trading system and is essential to integrating
WTO ministerial meeting developing members into global trade. S&DT provisions
are rights that must be preserved and strengthened in
both current and future WTO agreements.

o The World Trade Organization (WTO) is the only global


international organization dealing with the rules of trade
between nations.
WTO ministerial meeting of 16 developing and six least-developed
countries will be held in New Delhi. The main focus of this meeting is
Headquarters- Geneva, Switzerland
the appointment of appellate body members at the World Trade
Established in- 1995
Organisation and reforms at the global trade body with the aim, how
to constructively engage on various issues at both institutional and
negotiating.

o The World Trade Organization (WTO) is the only global international


organization dealing with the rules of trade between nations.
Headquarters- Geneva, Switzerland
Established in- 1995

Page 19
STUDY IQ
bankiq.in May 2019

Japan drags India to WTO In March, bowing down to international pressure, Pakistan
launched a major crackdown on Jaish-e-Mohammad, Jamat
Japan has dragged India to the World Trade Organisation (WTO) -ud-Dawa, and other banned outfits and took over the control
over the “continuously and systematically“ raise import duties on of their assets throughout the country.
certain electronic goods after the announcement of ‘Make in India’
campaign in September 2014.
It has also alleged that these import duties are in excess of bound
rates, which is a ceiling of import duty beyond which a WTO member
country cannot go.

India RCEP Trade

The Regional Comprehensive Economic Partnership


(RCEP) bloc companies 10 Asean group members and their
six FTA partners.
According to the provisional trade data, India’s trade
deficit (the difference between imports and exports) with
Pakistan airspace to remain shut for Indian Flights three countries (Brunei, Japan and Malaysia) has in fact
increasedmarginally in 2018-19 as compared to the previous
Pakistan on May 15 decided not to lift its airspace ban for Indian
fiscal.
flights till May 30, awaiting the outcome of the LokSabha polls in
Trade deficit with Australia, China, Indonesia, Korea, New
India. Pakistan fully closed its airspace after an Indian Air Force strike
Zealand and Thailand has narrowed in 2018-19 as compared
on a Jaish-e-Mohammed terror camp in Balakot on February 26. Now
to the preceding fiscal.
opened its airspace for all flights except for New Delhi, Bangkok and
Experts have mixed views over the impact of increasing
Kuala Lumpur .
trade gap on India’s position in negotiating mega free trade
The airlines and civil aviation authorities of both the countries are
agreement.
enduring massive losses.
An official said that India does not have free trade agreement
with two of its biggest trading partners- the US and China- but
Black money details the country has highest positive balance of trade with America,
while it has highest deficit with China.
The government has declined to share information on black RCEP negotiation started in Cambodian capital Phnom Penh
money cases received from Switzerland, citing confidentiality. Finance in November 2012 with the aim to cover goods, services,
Ministry said India and Switzerland share information on black money investments, economic and technical cooperation, competition
on a case to case basis as per the investigations being carried out. and intellectual property rights.
The information shared by Switzerland on black money cases is
governed by confidentiality provisions.
India and Switzerland are signatories to Multilateral Convention on
Mutual Administrative Assistance in Tax Matters (MAAC) on
November 22, 2016.

WTO consultations on India

Singapore and Canada have expressed interest in joining


consultations in a case filed by Japan in the WTO’s dispute
settlement body against India’s import duties on certain
information and communication technology products,
Questioning Pak against terror outfits
including mobile phones.
The European Union, China and Thailand have expressed
Pakistan’s seriousness to act against proscribed terror outfits and
interest to join consultations in a case filed by Japan at the
its efforts to curb money laundering and terror financing were
WTO against India’s import duties on certain information and
questioned by members of a regional affiliate of the Financial Action
communication technology products, including mobile phones.
Task Force (FATF) at a meeting held in China.
Bound tariffs or duties refer to the ceiling over which a WTO
India raised very tough questions about Pakistan’s seriousness.
member country cannot impose import duty, the applied tariff
Finance Minister ArunJaitley said India will ask the FATF to put
is the duty which is currently in place.
Pakistan on a blacklist of countries that fail to meet international
standards in stopping financial crime. Page 20
STUDY IQ
bankiq.in May 2019

Khadi Gram Udyog Bhawan is in Connaught Place, New Delhi.


Canada has said that during 2016-2018, India’s imports from
The business of khadi is Rs 1 crore or more a day at least three
Canada of ICT products have aggregated at USD 28.7 billion.
times between October and November. In 2017-18 the sales
worth a record of Rs 103 crore. In 2013-14 it was Rs 1,081
crore and in 2018-19 it is Rs 3,200 crore all-time high.
MSME ministry data shows sales stood at Rs 2,510 crore in
2017-18. As per First 4 years of Modi Govt. sales grew at an
annual average of over 30%, against 6.7% in the previous 10
Currency monitoring list years. In future some 83,000 postmen across states will be
clothed in khadi uniform.
US government has removed India from its monitoring list for
currency manipulation. This will hopefully set the tone for Notes
discussion of the Generalized System of Preferences (GSP) issue.
India was first included in the currency watch list in 2017 after the
Reserve bank of India has added $52 billion.

India Kyrgyzstan

President of Kyrgyzstan Sooronbay Jeenbekov, who holds the


current chair of the Shanghai Cooperation Organization, is a
surprise special guest at the swearing-in ceremony of Prime
Minister Narendra Modi.
Kyrgyzstan is sharing a request for defence equipment from
India, taking the strategic relationship beyond the current level of
bilateral exercises and the construction of a joint mountain
warfare training centre. India is likely to extend a line of credit to
Kyrgyzstan $100 million.
Bilateral relations have been on a strong footing since 2011 when
the joint ‘khanjar’ series of exercises started. Since then three IT-
Centres have been established by India.

Fishing & WTO

India is proposing to introduce a new discipline to put an end to


subsidies which lead to illegal, unregulated and uncontrolled
fishing globally. It has suggested capping ‘non-specific’ fuel
subsidies or those that are not targeted at a particular industry.
The US and European Union (EU) have opposed the proposal as
they give such subsidies to transport and water sports sector on the
other hand African, Arab and Latin American countries have
supported it.
Fuel subsidies comprise 60% of total fisheries subsidiesandalmost
95% of the fuel subsidies are nonspecific in developed countries.

Page 21
STUDY IQ
bankiq.in May 2019

Polity and
Notes

Governance

Niti Aayog’s economic agenda for new government

Niti Aayog is working on the economic agenda for the new


government for achieving long-term sustainable growth and
boosting private investments.
The economic agenda might also pitch for extending labour
subsidies to textile and leather sectors to reduce cost of production.
Cost of capital in India is quite high, which needs to be
brought down.

o The NITI Aayog is a policy think tank of the Government of India,


established with the aim to achieve Sustainable Development Goals
and to enhance cooperative federalism by fostering the involvement
of State Governments of India in the economic policy-making
process using a bottom-up approach.

S & P: Reform leading to improvement in


foreign capital flows

Modi’s decisive victory in 2019 Lok Sabha elections will result in


continuity and stability in the governments’ reforms policy,
leading to improvement in foreign capital flows for corporate in
India.
India’s bankruptcy code, simplification of the country’s tax
regime, and privatisation of inefficient state-owned enterprises
are all likely to create opportunities for private enterprises going
forward.

Page 22
STUDY IQ
bankiq.in May 2019

Appointments
Notes

and
Resignations

BSE Appoints First Independent Woman


Director JayshreeVyas To board

BSE, formerly known as the Bombay Stock Exchange, has


appointed its first independent woman director JayshreeVyas.
Vyas is a professionally-qualified chartered accountant, working as
the managing director of Shree MahilaSewaSahakari Bank,
Ahmedabad, since 1986.

o The Companies Act of 2013 mandates a certain class of


companies to have at least one woman director on board. Sebi, in
compliance with the Companies Act 2013, made it compulsory to
have at least one woman on board from October 2014.

BSE MD & CEO: AshishkumarChauhan

Page 23
STUDY IQ
bankiq.in May 2019

Government
o Unified Payments Interface (UPI) is an instant real-time
payment system developed by National Payments
Corporation of India facilitating inter-bank transactions. The
interface is regulated by the Reserve Bank of India.

Schemes
Introduced in- 11 April 2016

Notes

PM Kisan scheme

Modi government’s PM-Kisan scheme, since April 1, has benefited


a little over 2.25 crore farmers, including 1.08 crore in Uttar Pradesh
alone.
The farmers have received the second instalment of Rs 2,000 each
under the PM-Kisan scheme. However, another 86 lakh farmers, who
received only the first instalment of Rs 2,000, are still waiting for the
second instalment of Rs 2,000.
Agriculture ministry has firmed up a plan to extend benefits of the
PM Kisan scheme to cover an additional two crore farmers.

UPI

The value of transactions on the Unified Payments Interface (UPI)


platform recorded this January – Rs 1.09 lakh crore- overtook those
made by debit and credit cards at Rs 1.05 lakh crore.
UPI transactions are on the rise, consumers are increasingly using
net banking to make transactions of a large value, and therefore, the
average transaction value (ATV) for UPI transfers was small.
UPI usage shot up over the period between March 2018 and March
2019, with monthly transaction volumes growing 4.5 times to 799.54
million over the year.

Page 24
STUDY IQ
bankiq.in may 2019

Samsung sees a big opportunity for component business

Miscellaneous
considering the government’s thrust on ‘Make in India’ where
the tax on imported mobile phone components and
consumer electronics is going up. Samsung is also pitching
to the government for export incentives so that it can even
export components from India.
Samsung Display has already signed a MoU with the Uttar
Pradesh government for Rs 1,500 crore plant for
manufacturing mobile display.

o Make in India, a type of Swadeshi movement covering 25


sectors of the economy, was launched by the Government
of India on 25 September 2014 to encourage companies to
manufacture their products in India and enthuse with
dedicated investments into manufacturing.
Tough stand SA 701

The ministry of corporate affairs (MCA) has decided not to extend Meity to verify social media fake accounts
the deadline for applicability of Standard on Auditing SA 701 till
March 31, 2020, despite a demand from the Institute of Chartered The ministry of electronics and IT (MeitY) is exploring the
Accountants of India (ICAI). option of verifying user’s profile on social media platforms
The ministry has stated that “enough time” was given to auditors to such as Facebook, Whatsapp and Twitter which it feels could
familiarise themselves with SA 701, (first issued by the ICAI in May help check fake accounts that indulge in anti-social activities
2016 and was applicable for audit of financial statements after April 1, like spreading dubious information and propaganda. Individual
2017) which involves communicating key audit matters (KAM). online account of users of social media platforms should be
verified and fake ones weeded out.
UN Designates JeM Chief Masood Azhar
As Global Terrorist Whtaspp’s payments services and the
Supreme Court
UN designates Masood Azhar as global terrorist. In a huge
diplomatic win for India, the United Nations today designated WhatsApp’s payments services came under fire from the
Pakistan-based Jaish-e-Mohammed Chief Masood Azhar as a "global Supreme Court.
terrorist" after China lifted its hold on a proposal to blacklist him. The issue at stake is that overseas firms like WhatsApp need
to store all financial data within the country under a Reserve
o The UN Security Council has primary responsibility for the Bank of India notification. The deadline for meeting the norm
maintenance of international peace and security. for local storage of financial data ended on October 18, 2018.
o It has 15 Members, and each Member has one vote. Since WhatsApp’s payments services is still in pilot stage and
has not received the permission of National Payments
Corporation of India to go live, technically it is not ain violation
Samsung for Make in India of any guidelines.
The Supreme Court took note of a petition filed by an NGO
Electronic giant Samsung has lined up fresh investment of around Centre for Accountability and Systemic Change (CASC) which
Rs 2,500 crore to turn its India operations into a hub for has claimed that WhatsApp has not fully complied with the
components business. RBI’s circular which prescribed data localisation norms.
The Korean company has set up two new component RBI is free to prosecute it for failing to comply with data
manufacturing entities in India- Samsung Display Co localisation norms.
and Samsung SDI India- for production of mobile phone display and
batteries.
Page 25
STUDY IQ
bankiq.in May 2019

The next government may need to prime the pump as


CEA included in finance panel’s advisory council consumption has been one of the engines that has been
Chief Economic Advisor (CEA) Krishnamurthy Subramanian has driving the economy in the absence of private investment
been included in the Advisory Council (set up in April 2018) of the and exports.
15th Finance Commission (2020-2025) as its member. Passenger car volumes have dropped in nine of the past
The Advisory Council was set up to advice the commission on 10 months. Farm income growth has been weak for over
any issue or subject related to the terms of reference (ToR) of the two years with prices having stayed low.
commission, and also to assist in the preparation of any paper or
research study which would enhance the commission’s Services activity at 7 month low in
understanding on the issue contained in its ToR. April on slow sales

Reliance Jio to launch GigaFiber to improve call quality India’s service sector activity slipped to a seven month low
in April, due to weak rise in sales amid competitive pressures
Reliance Jio, which is working on the launch of its Fibre-to-home and disruptions arising from elections.
wirelineBroadband services JioGigaFiber. It will offer automatic Nikkei India Services Purchasing Managers’ Index (PMI)
routing of mobile voice services through landing networks, thereby dropped to 51 in April from 52 the previous month.
improving call quality by reducing call drops in a big way. The Nikkei India Composite PMI Output Index, which
Once JioGigaFiber is rolled out, subscribers will be able to set their combines both service and manufacturing, fell from 52.7 in
Jio mobile phones in a manner that once they are home, the voice March to 51.7 in April.
calls will get routed through their fibre-based landline network. It will Predictions that economic conditions will normalise after
work in exactly the same manner as today mobile data and Wi-Fi data the elections.
get synergised. The Delhi High Court’s recent stay on anti-profiteering
proceedings against Abbott Healthcare opening a window
for other firms to become party to the case that challenges
constitutional validity of the provision in the GST law, also
questioned the practice of anti-profiteering authorities to
expand the scope of investigation to multiple products of the
same company after completing probe on one.

Absence of a transparent liquidation policy

The absence of a transparent liquidation policy has resulted in


India accumulating foodgrains 2.7 times the buffer norm in its official
reserves after a gap of six years.
The foodgrains stock in the Central Pool was 56.82 million tonne
as of April 1, against the buffer norm of 21.04 million tonne.
Ashok Gulati, a former chairman of the Commission for Agricultural
Costs and Prices (CACP): “This is the height of economic inefficiency.
Chief Economic Advisor Krishnamurthy
Stocks need to be liquidated in the domestic market through Subramanian on NBFC crisis
open market sales scheme (OMSS) or exported”.
“The current system is extremely ad hoc, slow and costs the At present there is a mounting fears that more non-banking
nation heavily. A transparent liquidation policy is the need of hour, financial companies (NBFCs) may default like IL&FS due to a
which should automatically kick in when FCI is faced with surplus growing liquidity crunch.CEA said that the issue is actually
stocks than buffer norms. one of solvency of a very few players.
ASSET-LIABILITY mismatch of firms in the entire shadow-
banking space needs to be “very tightly and carefully
India’s consumption story losing the plot
monitored” to ensure the crisis doesn’t recur or flare up.
Subramanian said the economy will likely grow at 7%-plus in
Consumption is sputtering across a range of products including
the current fiscal, against the estimated 7% for FY19. Structural
cars, two-wheelers,air travel and fast-moving consumer goods
reforms like GST and Insolvency and bankruptcy code (IBC)
(FMCG), with volumes dropping to multi-quarter lows.
have already been undertaken. He prescribed the option of a
The decline in demand stems from an income growth slump in
seven-day default rule for lenders if the one-day default rule
urban and rural areas that’s forced people to curb spending,
is too difficult to be implemented.
falling moneysupply in the economy and rising uncertainty over
how customers will respond to regulatory action.

Page 26
STUDY IQ
bankiq.in May 2019

Jet Airways crisis Shutting down ATMs

The ministry of corporate affairs (MCA) is understood to have asked Finding an ATM in India is getting tougher even as
the Serious Fraud Investigation Office (SFIO) to begin a probe into dependence on cash persists. Tighter regulations are
Jet Airways on suspicion that the promoters siphoned off money. making it more costly to run the machines.
The move follows after the MCA started a preliminary enquiry into According to the IMF, India already has the fewest ATMs
the company by inspecting its books. The MCA also want a probe as per 100,000 people among BRICS nations.
to how Jet suddenly turned in a loss in FY18 after posting a profit for The drop may continue as banks and ATM operators
a couple of years. struggle to absorb the cost of software and equipment
upgrades. ATM operators which include banks as well as
third parties charge a so-called interchange fee of 15 rupees
to the lender whose debit or credit card is used for cash
withdrawals. Interchange fees are the biggest factor
behind muted growth of ATMs.
The declining number of ATMs is likely to further boost
mobile banking,grew 65 times in the past five years alone.
Founder- Shaktikanta Das
Founded in- Mumbai, Maharashtra

Android abuse

Competition Commission of India has ordered an investigation


into Google for allegedly abusing the dominant position of its popular
Android mobile operating system to block rivals.
The CCI last year started looking into the compliant, which is similar
to the one Google faced in Europe that resulted in a $5-billion fine on
the company.
The probe will be completed in about a year and Google executives
will likely be summoned to appear before the CCI in coming months. Exports & Trade Deficit

India’s export growth slid to a four-month low of 0.64% in


April as shipments of engineering goods, gems & jewellery,
leather and other products declined, widening the trade
deficit to a five-month high.
Imports increased by 4.5% to $41.4 billion, the highest
growth in the last six months.
Google Headquarter- California, US
Founded in- 1998 India’s digital economy
CEO-Sundar Pichai
India is taking a great digital leap.
Ease of Doing Business The country is now seizing new digital opportunities in
many more sectors, such as agriculture, education, energy,
In an attempt to bring down the compliance burden on financial services, health care, and logistics, which could
companies, especially the small and medium enterprises (SMEs), the deliver up to $500 billion of economic value by 2025.
commerce ministry has urged the ministry of corporate affairs (MCA) India’s digitisation process has been the second-fastest
to explore avenues where firms can opt for self-certifying their among the 17 mature and emerging economies. It has 560
returns as well as clubbing all the returns to be filed in a year into million Internet subscribers.
a single annual return. Indians downloaded 12.3 billion apps, second only to the
A commerce ministry official said that the government is committed Chinese. Indians used more than 54 times as much data, on
towards creating a conducive business environment by streamlining average, in 2018 than in mid-2016.
such regulatory structures. “Moving away from profession certifications Both the public and private sectors have played an important
to self-certification would greatly increase the risk of fraud, wilful role in driving digitization.
misstatement and suppression of facts leading to corporate 1.2 billion Peoplenow enrolled with Aadhaar. Many public
mismanagement”. services are now accessible only when linked to the
government’s Aadhaar biometric digital-identification program.
Page 27
STUDY IQ
bankiq.in May 2019

The private sector has facilitated this process, as competition has AI in MCA 21 portal
helped to reduce data costs by 95% from 2013 to 2017 and to make
smart phones affordable.
Corporate affairs ministry plans to introduce artificial
In financial services, the surge in digital payments and related data
intelligence system in the MCA 21 Portal as it seeks to make
is already enabling flow-based lending whereby actual patterns of
compliance process easier as well as ensure routine
receipts and payments rather than loan applications are used to
enforcement activities are done round-the-clock on
evaluate potential borrowers.
autopilot basis.
We estimate that, by 2025, technology could estimate between
MCA 21 is the electronic backbone for dissemination of
40-45 million mostly routine jobs in areas such as clerical services
information to all stakeholders, including the regulator,
and data entry. It will also help to create some 60-65 million higher-
corporate and investors.
quality jobs.

Regulatory framework

TRAI will decide whether over-the-top (OTT)communication


applications like WhatsApp, Skypeetc, should be brought under
a regulatory framework.If it decides to bring them then these
apps will have to get licences from the government to operate
Digital India launched in- 1 July 2015 just as telecom operators have to currently do.
Telecom service providers (TSPs) are governed by a definite
set of obligations under their licensing conditions, no such
CERC for Power Companies obligations are applicable on OTTs.

Central Electricity Regulatory Commission (CERC) has allowed TRAI formed on- 20 February 1997
power companies to claim compensation for the additional cost
of coal procured from alternative sources due to Coal India’s New standards for measuring
failure to meet supply obligations. Such compensation facility was
available for the FY14-FY17. India adopted a global resolution to redefine four of the seven
The compensation on account of coal shortage would be worked base units. Four of the seven base units passed by
out according a formula prescribed by the CERC on a case to case representatives of 60 countries at the General Conference on
basis. Weights and Measures (CGPM).
As per the modified fiats of the New Coal Distribution Policy Kilogramme, Kelvin, Mole, Ampere.
(NCDP), power plants receive 75 per cent of their contracted fuel It has been implemented across the world on May 20- World
quantities through CIL linkages. Meteorology Day.
The purpose of a system unit is to enable worldwide
Refurbished mobile coherence of measurements.

India has finally allowed import of refurbished or second mobile MSDE on Reforms
phones, subject to the condition that they are certified by the
Bureau of Indian Standards (BIS). Ministry of Skill Development and Entrepreneurship’s top
Though the move meets demands of mobile operators like Apple, agenda for next government is to-
which has been seeking permission for import of refurbished 1. Operationalisation of the skills regulator,
iPhones for the past few years, analysts said the process is 2. An enhanced apprenticeship programme
cumbersome and fraught with risks. 3. Rejuvenation of 2,500 ITIs.
The government was earlier against import of refurbished To impart improved training to make youth job-ready.
electronics products into the country, fearing that companies may Last October, cabinet okeyed National Council for Vocational
use it to dump hazardous electronic waste. It would negate the effect Education and Training (NCVET). NCVET will regulate the
of Make in India initiative. functioning of entities engaged in vocational education and
training, and establish minimum standards for them.
States should contribute It will also focus on research, information dissemination and
grievance redressal.
State governments should contribute in funding for defence, The ministry will have to work with the states and attract
national highways and railways projects, said BibekDebroy, public investments in Industrial Training Institutes to make
Chairman of Economic Advisory Council to Prime Minister (EAC-PM). them world-class.
Fiscal consolidation is important, he said there is a need to prioritise The aim was to impart skills training to 400 million people by
and decide areas that are important from point of view of public 2022 through flagship schemes such as PradhanMantriKaushal
administration.He said as long as we have tax exemptions we will VikasYojana, the DeenDayalUpadhyayaGrameenKaushalyaYojana.
not have simplified tax structure.
STUDY IQ Page 28
bankiq.in May 2019

TPCI on RCEP The seven industry specific suggestions include separate


regulation for medical devices and a single ministry for the
India’s economic growth will regain strength and approach 7.5% sector.
by 2020 buoyed by rural consumption and subdued inflation, the For textiles and garments sector, it suggested modification
Organisation for Economic Co-operation and Development in Labour laws to remove limitation on firm size and allow
(OECD) said in its Economic Outlook. manufacturing firms to grow, also to promote tourism and
This growth will come from higher domestic demand due to medical value tourism.
improved financial conditions, fiscal and quasi-fiscal stimulus, To promote agriculture exports, it has asked for abolishing
including new income support measures for rural farmers, and Essential Commodities Act and the APMC (Agricultural
recent structural reforms. Produce Market Committee).

Energy goals for poorest nations

Report produced by the World Bank and other international


bodies like International Energy Agency, World Health
Organization, United Nations Statistics Division, International
Renewable Energy Agency.
More than 150 million people are gaining access to
Individual insolvency
electricity every year, but this is not enough to meet global
development goals.
Government is preparing individual insolvency regulations
In 2015, all United Nations member states adopted 17 goals
under the Insolvency and Bankruptcy Code (IBC).
for sustainable development by 2030 in areas such as education,
It will provide for debt waiver
climate, and gender equality as well as access to energy.
o Up to Rs 35,000
With 11 years to go, signs still point to failure. If governments
o To the poorest of the poor
do not make faster progress, 650 million will still live in the dark
o Who have borrowed money from not just banks but also informal
by 2030,90 percent of whom will be in Sub-Saharan Africa.
sources like village money lenders
Once the regulations are in place,
o the poor who don’t own houses,
1,000-crore fund for start-ups
o earn up to Rs 60,000 a year
o have assets up to Rs 20,000 will be eligible to apply for such a relief. The government proposes to introduce a slew of reforms
The regulations will be among the first set of measures to be and another fund to boost start-ups that are focused on
implemented under the new government. priority areas such as
Government believes it will be more effective than populist o Rural healthcare
moves like farm loan waivers that involve relief from just bank debt o Water and waste management
and is mostly exploited by rich farmers. o Clean energy solutions
o Cyber security and drones
DPIIT plans to set up an India Start-up Fund with an initial
Elephant Bonds
amount of Rs 1,000 crore.

In September last year, a 12-member group, was set up by the


Commerce Ministry, suggested issuance of Elephant Bonds also
recommended a host of other measures.
Elephant Bonds: people declaring undisclosed income will have
to mandatorily invest half of that amount in these securities.
Increasing India’s exports of goods and services from USD 500
billion is 2018 to over USD 1000 billion in 2025.
Lowering effective corporate tax rate.
Simplifying regulatory and tax framework for foreign ArvindPanagariya on new REFORMS
investments funds. The group recommended increasing capital
base of EXIM Bank by another Rs. 20,000 crore by 2022. Economy is the issue number one and that impacts
Setting up of empowered investment promotion agency and everything else, even our international standing gets
seeking inputs from industry and MSMEs before signing free trade influenced by how well the economy does. I am pretty
agreements (FTAs) and sensitising them of its benefits. confident that the new government will move on several
“State governments need to be closely involved in improving the fronts as soon as the cabinet is in place.
competitiveness of exports by providing support measures in a
WTO (World Trade Organisation) consistent manner”.
Page 29
STUDY IQ
bankiq.in May 2019

He said, it would be good if the government in its first 100 days Excess’ stocks of FCI creates huge losses
gives a very strong signal for its commitment to reforms.
The final budget will be presented probably in July, maybe even in Till 1st April 2019 FCI & state government agencies were
June, and/or even before that a number of things could be holding excess food grain stock worth Rs 1.18 lakh crore.
announced including the corporate profit tax. This was even as fiscally-stressed Centre’s food subsidy dues
The government could also give a signal by rolling back many of to FCI were close to a staggering Rs 2 lakh crore at the start of
the tariffs that had been raised in the past two years. The government this fiscal and for the third year in a row.
can do is to also speed up privatisation, It is time to actually carry FCI borrowed an additional of Rs 60,000crore from the
out that mandate. NSSF in April 2019 to ensure its operations under the National
NPA cleanup needs to be speeded up. Food Security Act are not disrupted.
The industry needs to shape up a little better and I have never While offloading the excess stocks appears to be a rational
understood this whole talk about consumption slowing down, option for the FCI rather than pile up more debt, it lacks a
because the global economic is large. There were interest rates policy mandate to exercise that choice.
complaints, particularly auto when you say for 70 years they have Even if the FCI were to sell the excess stock in the open market
been protected with 100 plus kind of tariffs. Automobiles are now, it would fetch some Rs 30,000 crore less than the cost
paying a punishing kind of price for automobiles. incurred by it to create it.
The wages in China have really gone up now and our wages
significantly lower and that alone gives us a big advantage in large CII demands for lowering corporate tax rate
number of labour-intensive industries.
The trade war between the United States and China have deep Industry body Confederation of Indian Industry (CII) has called
roots and therefore it is not going to go away so quickly and that for lowering corporate tax rate, kick-starting government
does give us a major opportunity to break into both the American expenditure and rationalisation of dispute tax resolution
market because Chinese now are facing higher barriers in the mechanism. Dividend distribution tax should be rationalised
American market and in the Chinese market because now the to 10% (from 15% currently).
Americans are facing high barriers in the Chinese market. We have got Long-term capital gains tax on equities and MAT should
a double opportunity there. be removed.
I found this repeatedly in my experience at the NITI Aayog that the Government stake in public sector banks should be reduced
ministries do not work in national interest, they work in their from 70% currently to 51% to enable capital infusion and
narrow interest of the ministry itself. promote efficiency in public sector banks.

DPIIT for shortening compliance time to just 1 378 companies under IBC
hr/ month for start-ups
As many as 378 companies with total creditor claims of Rs
The Commerce and Industry Ministry has proposed cutting down 2,57,642 crore have so far been sent into liquidation under
compliance time significantly to just one hour per month for start- the Insolvency and Bankruptcy Code.64 companies, or
ups as part of measures to ease regulatory requirements for budding more than 16%, had received bids higher than the liquidation
entrepreneurs. value of the assets.
The proposal is a part of ‘Start-up India Vision 2024’, prepared by Liquidation value is the estimated realisable value of the
the Department for promotion of Industry and Internal Trade (DPIIT) assets of a corporate debt or if it is liquidated at the
for the new government to promote the growth of budding beginning of the insolvency proceeding.
entrepreneurs. At present, start-ups comply with a plethora of Based on the liquidation value, creditors to the companies
requirements such as GST filings, tax returns and other local laws every sent into liquidation would on an average recover only 7.1%
month. Compliance to these processes takes a lot of time and cost. of their admitted claims.
At present, start-ups comply with a plethora of requirements such IBBI data also showed that in the 88 cases of successful
as GST filings, tax returns and other local laws every month. resolutions under the bankruptcy law since it came to effect
Compliance to these processes takes a lot of time and cost. in December 2016, operational creditors and financial
Monthly compliance for start-ups needs to be reduced to one hour creditors recovered about 48% of their claims.
per month so that they can concentrate on their core work.
The vision document has suggested some important measures-
o Setting up of a regulatory sandbox or innovation hub to help
fintech start-ups;
o Tax incentives for investments in ventures of budding entrepreneurs;
o Reduction in GST rates on alternate investment fund management services;
o Amendment in income tax laws pertaining to sale of residential
properties and carrying forward of losses.
o Deployment of entire corpus of Rs 10,000 crore Funds of Funds.
Page 30
STUDY IQ
bankiq.in May 2019

100-day agenda for the new Government


Notes
The commerce and industry ministry has proposed creation of a
separate department for trade facilitation and logistics for better
coordination among different government agencies.
In its 100-day agenda for the new government, the ministry said,
“Economic Advisory Council to the Prime Minister (PMEAC) has also
recommended the same.
India’s logistics and transportation costs are pegged at 14.4% of the
gross domestic product, much higher than China’s 8%.
Multi-Modal Transportation of Goods (MMTG) Bill will be introduced
in Parliament to replace the existing MMTG Act, 1993.

Index to rank states on artificial intelligence adoption

Artificial intelligence (AI) readiness index by NITI Aayog that will


rank states on their capacity to adopt the technology for public
service delivery and exploit its innovative potential.
One of the main roles of NITI is to promote competitive and
cooperative federalism.
AI’s potential is $957 billion to India’s GDP growth & 1.3 percentage
points by 2035.

o The NITI Aayog is a policy think tank of the Government of India,


established with the aim to achieve Sustainable Development Goals
and to enhance cooperative federalism by fostering the involvement
of State Governments of India in the economic policy-making
process using a bottom-up approach.

World-class infrastructure club

A task force on project management led by Niti Aayog chief


executive Amitabh Kant has pitched for a dedicated policy
frameworkfor public-private partnerships and public sector projects
to improve efficiency.
National project/ Programme Management Policy Framework
(NPMPF) should be made part of all future contracts so that we can
create world class infrastructure.
The task force has also recommended a dedicated workforce be
developed with expertise in smooth rollout, implementation and
completion of all projects, much on the lines of the US and UK, to
assist in project implementation.

Did you know how the Indian Rupees are made?

The Indian Rupee note made up of Cotton and Cotton rag.


Technically, it is not a paper but it looks and feel of paper.
Page 31
STUDY IQ
bankiq.in MAY 2019

PRACTICE CURRENT AFFAIRS MCQ’S

BANKING & FINANACIAL AWARENESS


Q1) Securities and Exchange Board of India (SEBI) has barred National Stock Exchange (NSE) from
accessing the securities market directly or indirectly for a period of ___________ ?

(a) 3 months (b) 6 months (c) 7 months (d) 5 months

Q2) Which Bank will link its interest rate on savings account with balance above Rs. 1 lakh
to Reserve Bank of India’s repo rate, effective 1 May 2019 ?

(a) Punjab National Bank (b) Bank of Baroda (c) State Bank of India (d) Indian Bank

Q3) Which country was the biggest recipient of funds from the Asian Development
Bank (ADB) in the year 2018?

(a) Armenia (b) India (c) Hong Kong (d) Fiji

Q4) Reserve Bank of India has imposed a penalty of Rs 11.25 lakh on which bank for
violating money transfer norms?

(a) ICICI Bank (b) Yes Bank (c) Federal Bank (d) HDFC Bank

Q5) The Reserve Bank of India has imposed penalties on which prepaid payment
instrument (PPI) issuers for violation of regulatory norms?

(a) Vodafone m-pesa (b) PhonePe (c) Western Union (d) All of the above

Q6) The World Bank ha announced a 545 million U.S. dollars donation to help people in
Mozambique, Malawi, and Zimbabwe that have been affected by which cyclone recently ?

(a) Pabuk (b) Idai (c) Veronica (d) Mona

Q7) According to latest data released by RBI India’s Foreign Exchange Reserves has
risen by_____ Billion dollars as on April 26?

(a) $ 5 Billion (b) $ 3.2 Billion (c) $ 4.3 Billion (d) $ 4.8 Billion

Q8) Which country’s 10 dollar Bill/Currency has been adjudged as the best bank note
for 2018 by the International Bank Note Society (IBNS)?

(a) USA (b) Canada (c) France (d) Sweden

Q9) Asian Development Bank (ADB) has launched __________billion dollar Action Plan for Healthy
Oceans and Sustainable Blue Economies for the Asia and Pacific region at its 52nd Annual Meeting?

(a) Billion Dollar (b) 4 Billion Dollar (c) 5 Billion Dollar (d) 2 Billion Dollar

Q10) Which country’s central bank has printed the $50 note with a spelling mistake where the
word “responsibility”is printed as “responsibilty”?

(a) USA (b) Canada (c) Germany (d) Australia

Page 32
STUDY IQ
bankiq.in MAY 2019

APPOINTMENTS & RESIGNATIONS


Q1) Who will take over as the Vice Chief of Indian Air Force from May 1, 2019?

(a) Anil Khosla (b) Rakesh Kumar Bhadauria (c) Surya Singh Prakash (d) Narendra Singh Rajput

Q2) Who has been appointed as the Officer on Special Duty (OSD) in the office of Lokpal ?

(a) Ramesh Singh (b) Vihan Thakur (c) Dilip Kumar (d) Pinaki Chandra Ghose

Q3) General S.H.S Kottegoda has been appointed as the Defence Secretary of which country?

(a) Indonesia (b) Nepal (c) Sri Lanka (d) DBhutan

Q4) Q4) Name the President of the Archery Association of India who has given his resignation?

(a) Ajay Singh (b) B.V.P. Rao (c) Narinder Dhruv Batra (d) Ajay Rastogi

Q5) BSE, formerly known as the Bombay Stock Exchange, has appointed whom a
s its first independent woman director?

(a) Niranjana Jha (b) Jayshree Vyas (c) Kashvi Patel (d) Archana Baghel

Q6) Who has been appointed as member of Advisory Council of 15th Finance Commission ?

(a) Krishnamurthy Subramanian (b) Arvind Subramanian (c) Arvind Virmani (d) Kaushik Basu

Q7) Bhushan Patil who has given his resignation was the President of which company?

(a) Flipkart (b) Amazon (c) Google India (d) Paytm

Q8) Godavarthi Venkata Srinivas was appointed as the next Ambassador of India to
which of the following countries?

(a) Gambia (b) Mauritania (c) Senegal (d) Mali

Q9) Who has been appointed as the Governor of State Bank of Pakistan?

(a) Kassim Parekh (b) Salim Raza (c) Yaseen Anwar (d) Reza Baqir

Q10) Who has been appointed as the new Chairman and Managing Director of ITC?

(a) Vineet Taneja (b) Pradeep Jain (c) Pawan Munjal (d) Sanjiv Puri

Q11) Who was appointed as a member of Commonwealth Secretariat Arbitral Tribunal (CSAT)?

(a) KS Radhakrishnan (b) Madan Lokur (c) H L Dattu (d) Dinesh Maheshwari

Q12) Who has been appointed as the new President of the United Nations General Assembly?

(a) Tijjani Mohammad Bande (b) María Fernanda Espinosa (c) Joseph Nanven Garba (d) Peter Thomson

Q13) Cooperative major IFFCO has elected whom as its chairman?


(a) Balvinder Singh Nakai (b) Dileep Sanghani (c) Vinod Kumar Goyal (d) Pratap Singh
Page 33
STUDY IQ
bankiq.in MAY 2019

Days and Dates

Q1) International Dance Day was observed on?

(a) A. April 30 (b) April 28 (c) April 27 (d) April 29

Q2) Ayushman Bharat Diwas was celebrated across the country on?

(a) 27April (b) 29 April (c) 30 April (d) 28 April

Q3) What was the theme of the International Labour’s Day?

(a) Uniting Workers for Social and Economic Advancement (b) Workers are backbone of the society
(c) World is nothing without Labours (d) None of the above

Q4) World Press Freedom Day was observed on?

(a) May 1 (b) May 2 (c) May 3 (d) April 30

Q5) National Space Day 2019 was observed on?

(a) May 1 (b) May 2 (c) May 3 (d) May 4

Q6) Coal Miners Day 2019 was observed on ?

(a) May 4 (b) May 3 (c) C May 2 (d) May 1

Q7) World Laughter Day 2019 was observed on?

(a) May 5 (b) May 4 (c) May 3 (d) May 2

Q8) World Asthma Day 2019 was observed on?

(a) May 4 (b) May 6 (c) May 7 (d) May 3

Q9) World Migratory Bird Day 2019 was observed on?

(a) May 10 (b) May 9 (c) May 11 (d) May 8

Q10) National Technology Day in India was observed on?

(a) May 11 (b) May 10 (c) May 9 (d) May 8

Q11) The World Athletics Day 2019 was observed on?

(a) May 8 (b) May 7 (c) May 9 (d) May 6

Page 34
STUDY IQ
bankiq.in MAY 2019

Defence & security


Q1) The Indian naval ships INS Kolkata and INS Shakti have reached which country
to participate in the three days ADMM-Plus Maritime Security Field Training Exercise (FTX)?

(a) Russia (b) Japan (c) Iran (d) South Korea


Q2) Ministry of Defence (MoD) is going to establish which agency by May to tackle
non-civilian cyber issues arise from China and Pakistan?

(a) Special Cyber Agency (b) Defence Cyber Agency (c) Space Cyber Agency (d) Armed forces Cyber Agency

Q3) The 17th edition of the joint naval exercise, “Varuna-19”, has started
between India and which country?

(a) Russia (b) Singapore (c) France (d) Indonesia

Q4) Indian Navy has launched the fourth Scorpene Class Submarine named what?

(a) INS Vela (b) INS Kalvari (c) INS Kursura (d) INS Karanj

Honours and Awards

Q1) Who was awarded the Goldmanal Prize?

(a) Alberto Curamil (b) Jacqueline Evans (c) Alfred Brownell (d) All of the above

Q2) Which country has honoured Lt. Gen. (retd) Jack Farj Rafael (JFR) Jacob with a plaque on
Ammunition Hill Wall of Hon

(a) UAE (b) Saudi Arab (c) Israel (d) Australia

Q3) Who was conferred with France's highest civilian honour, Chevalier de l'Ordre
national de la Legion d'Honneur?

(a) A S Kiran (b) K Sivan (c) G. Satheesh Reddy (d) Avinash Chander

Q4) Who has been declared as the top military officer (Supreme Allied Commander Europe)
of the 29-nation NATO military alliance?

(a) Richard Abel (b) James Abrahamson (c) Tod D Wolters (d) Curtis M. Scaparrotti

Q5) The article titled Coalgate 2.0' authored by whom has won the ACJ Award for
Investigative Journalism, 2018?

(a) Nileena M S (b) Shereen Bhan (c) Ashok Malik (d) Karma Paljor

Page 35
STUDY IQ
bankiq.in MAY 2019

Q6) Who was awarded with the 27th P.C. Chandra Puraskaar?

(a) Devi Prasad Shetty (b) Surender Nath Khanna (c) Ramakanta Panda (d) Amit Mittal

Q14) Who has taken charge as India's new envoy to China?

(a) VikramMisri (b) Sanjay Kumar Verma (c) RuchiGhanshyam (d) RanjitSethi

Q7) Which two Indians are included among 115 UN personnel staff honoured for sacrifice in line of duty?

(a) Radhika Singh, Suresh Goyal (b) Jitender Kumar, Shikha Garg (c) Surbhi Sharma, Subodh Verma (d) Anshul Patel, Vinital Messy

Q8) Who has been awarded the 2019 V K Krishna Menon award posthumously for his outstanding
contribution in journalism??

(a) Satyendra Murli (b) G D Robert Govender (c) Gour Kishore Ghosh (d) Gaurav Smith Arya

Q9) Who was honoured with the 2019 McCain Institute Award for Courage and Leadership?

(a) Chhaya Sharma (b) Archana Ramasundaram (c) Meera Borwankar (d) Vimla Mehra

Q10) Who has been awarded the 2019 V K Krishna Menon award posthumously for his
outstanding contribution in journalism?

(a) Satyendra Murli (b) G D Robert Govender (c) Gour Kishore Ghosh (d) Gaurav Smith Arya

Notes

Page 36
STUDY IQ
bankiq.in MAY 2019

India and the World

Q1) Which country has emerged as the third largest foreign direct investor in India during 2017-18?

(a) Netherlands (b) Norway (c) Denmark (d)Hong Kong

Q2) The Government has extended its deadline to impose retaliatory import duties on how
many products from the United States (US), including almond, walnut and pulses, till May 16 ?

(a) 20 (b) 16 (c) 29 (d) 25

Q3) The 11th meeting of the joint working group on counter-terrorism between India and
Australia held at which city?

(a) New Delhi (b) Sydney (c) Canberra (d) Amravati

Q4) The United Kingdom is in talks with the Indian government on building a new state-of-the-art
aircraft carrier named what?

(a) INS Rohit (b) INS Vishal (c) INS Samudra (d) INS Kirti

Industry

Q1) Vodafone Idea Ltd has signed an agreement with which IT company?

(a) Wipro (b) IBM (c) Infosys (d) TCS

Q2) State-owned Oil and Natural Gas Corporation (ONGC) has received the green nod to drill
6 development wells in which state at an estimated cost of Rs 240 crore?

(a) Manipur (b) Tripura (c) Assam (d) Nagaland

Infrastructure

Q1) LIC Housing Finance (LIC HFL) has launched ‘Udyam’, a skilling centre in which city?

(a) Bengaluru (b) Nasik (c) Chennai (d) Indore

Page 37
STUDY IQ
bankiq.in MAY 2019

International Affairs
Q1) Which country is set to withdraw from the U.N. Arms Trade Treaty?

(a) Australia (b) China (c) Canada (d) US

Q3) Stevo Pendarovski has won the run-off Presidential elections conducted in which country?

(a) North Macedonia (b) Greece (c) Albania (d)Serbia

Q4) Which country’s Air Force has successfully tested laser weapons system called ‘SHiELD’ to
shoot down Missiles?

(a) Russia (b) China (c) USA (d) Japan

Q5) Pakistan will receive $ _________ billion from International Monetary Fund for over three years?

(a) 3 billion (b) 4 billion (c) 5 billion (d) 6 billion

Q6) Which country has tested the world’s fastest bullet train called ALFA-X with Speed of 400 kmph?

(a) China (b) Russia (c) Japan (d) USA

Q7) Laurentino Cortizo has won the presidential elections of which country?

(c) Costa Rica (b) Angola (c) Panama (d) Comoros

Notes

Page 38
STUDY IQ
bankiq.in MAY 2019

Polity and Governance

Q1) The Supreme Court has rejected the review petition filed by 21 opposition parties seeking ______
percent verification of Voter Verified Paper Audit Trail (VVPAT) during counting of votes in Lok Sabha
Elections 2019 ?

(a) 40 % (b) 100 % (c) 70 % (d) 50 %

Q2) The Election Commission of India has permitted relaxation in the model code of conduct for
which of the following states?

(a) Karnataka (b) Andhra Pradesh (c) Odisha (d) Maharashtra

Reports and Indices

Q1) According to a latest report, Drug-resistant diseases could cause 10 million deaths each
year by which year?

(a) 2030 (b) 2040 (c) 2050 (d) 2060

Q2) According to a latest report from the Stockholm International Peace Research Institute (SIPRI),
India was the world’s ________largest military spender in the year 2018?

(a) Fifth (b) Fourth (c)Third (d) Sixth

Q3) India Ratings and Research has predicted India’s GDP growth rate prediction
for 2019-20 at what percent?

(a) 7 % (b) 7.4 % (c) 7.1 % (d) 7.3%

Q4) According to a report titled ‘Cyber Insurance in India’ released by the Data Security Council of India
(DSCI), Indian cyber insurance market has grown to what percent more in 2018 as against 2017?

(a) 30 % (b) 25 % (c) 50 % (d) 40 %

Q5) According to a new report, which city has emerged as the top choice for Indian investors with
record investments in 2018?

(a) Dubai (b) Singapore (c) London (d) Tokyo

Page 39
STUDY IQ
bankiq.in MAY 2019

Sports

Q1) Which F1 racer has won the 2019 Azerbaijan Grand Prix?

(a) Lewis Hamilton (b) Sebastian Vettel (c) Valtteri Bottas (d)Sergio Perez

Q2) Who has clinched the women’s singles title at Stuttgart’s Porsche Grand Prix 2019?

(a) Petra Kvitova (b) Simona Halep (c) Naomi Osaka (d) Angelique Kerbe

Q3) Who is set to become the first wrestler from India to fight at the iconic Madison Square
Garden in New York?

(a) Rahul Aware (b) Deepak Punia (c) Praveen Rana (d) Bajrang Punia

Q4) Who has been named the 2019 footballer of the year by the Football Writers' Association (FWA)?

(A) Raheem Sterling (b) Virgil Van Dijk (c) Lionerl Messi (d) Luka Modric

Q5) Who has become the first Indian to rank in the top 25 rankings of ITTF rankings?

(a) G Sathiyan (b)Sharath Kamal (c) Ramkumar Ramanathan (d) Prajnesh Gunneswaran

Q6) Who has become the first woman to referee a French Ligue 1 match?

(a) Stephanie Frappart (b) Bibiana Steinhaus (c) Hope Solo (d) Lucy Bronze

Q7) Which football club has won the Spanish La Liga title 2019?

(a) Barcelona (b) Levante (c) Liverpool (d) Chelsea

Q8) Who has been appointed as the first non-British president of the Marylebone Cricket Club ?

(a) Kumar Sangakkara (b) Rahul Dravid (c) Sanath Jayasuriya (d) Kapil Dev

Q9) Who has attained the world number one position in the women's 10 metres air rifle event ?

(a) Manu Bhaker (b) Anjum Moudgil (c) Apurvi Chandela (d) Heena Sidhu.

Q10) Table Tennis Federation of India (TTFI) has decided to bid for which edition of the biennial
World Team Championships?

(a) 2022 (c)2024 (c)2026 (c) 2028

Q11) Which Indian wrestler bagged gold medal in the 2019 Ali Aliyev Wrestling Tournament ?

(a) Sushil Kumar (b) Bajrang Punia (c) Gurpreet Singh (d) Harpreet Singh

Page 40
STUDY IQ
bankiq.in MAY 2019

Q12) Which of the following player has received the Orange Cap for scoring maximum
runs at the IPL 2019?

(a) Imran Tahir (b) David Warner (c) Andre Russell (d) K L Rahul

Q13) Which female cricketer has become the highest wicket-taking spinner in ODI?

(a) Jhulan Goswami (b) Sana Mir (c) Suzie Bates (d) Meg Lanning

Q14) Who has become the third Indian to claim 150 wickets in the Indian Premier League (IPL)?

(a) Piyush Chawla (b) Amit Mishra (c) Mohit Sharma (d) Harbhajan Singh

Q15) Who has been appointed as India men’s football team coach?

(a) Jim Harbaugh (b)Tony Dungy (c) Marv Levy (d) Igor Stimac

Notes

Page 41
STUDY IQ
bankiq.in MAY 2019

Miscellaneous

Q1) Shanghai Cooperation Organisation (SCO) Defence Ministers’ Meet was organized in which country?

(a) Tajikistan (b) China (c) Kazakhstan (d) Kyrgyzstan

Q2) Which company has launched a new option of recurring payments for its merchants ?

(a) Google Pay (b) PhonePe (c) Paytm (d) Samsung Pay

Q3) Which General Insurance Company has become the first in the Indian market to collaborated with
web aggregator, Wishfin Insurance to sell 2-wheeler policies via WhatsApp ?

(a) Bajaj Allianz (b) Bharti AXA (c) HDFC Ergo (d) Max Bupa

Q4) B Subhashan Reddy who passed away recently was a former Chief Justice of which High Courts?

(a) Madras (b) Guwahati (c) Delhi (d) Patna

Q5) Government of India has made it mandatory for Electric Vehicles to have number/license
plates in which Color ?

(a) Blue (b) Black (c) Red (d) Green

Q6) India is planning to launch its latest radar imaging satellite named “what” towards the end
of May 2019?

(a) GISAT 2BR1 ( b) VISAT 2BR1 (c) KISAT 2BR1 (d) RISAT 2BR1

Q7) The Police of which city has launched an all-women police patrol unit ‘Rani Abbakka Force’ ?

(a) Chennai (b)Bengaluru (c) Mangaluru (d) Kolkata

Q8) The 16th Ministerial Meeting of Asia Cooperation Dialogue (ACD) was held in which country ?

(a) UAE (b) Qatar (c) Saudi Arab (d) India

Q9) India will purchase 10 Kamov-31 choppers from which country for 3,600 crores ?

(a) France (b) Israel (c) Russia (d) USA

Q10) The Indian Space Research Organization is planning to send a probe to study the sun early
in 2020. The name of the mission is ?

(a) Aditya-L1 (b) Aditya-K1 (c) Aditya-G1 (d) Aditya-L2

Q11) Which of the following has launched a new initiative named Global Registry of Violent Deaths
(GReVD) to establish the annual number of violent deaths worldwide?

(a) UNHRC (b) SIPRI (c) WHO (d) UNDP

Page 42
STUDY IQ
bankiq.in MAY 2019

Q12) “Game Changer” is the Autobiography of which player?

(a) Shoaib Akhtar (b) Shahid Afridi (c) Shoaib Malik (d) Wasim Akram

Q13) The Group of 7 (G7) industrialised nations meet is going to be organized in August 2019
in which country?

(a) Italy (b) France (c) Japan (d) Germany

Q14) Konkan Railways Corporation Ltd has signed a contract agreement with which country to
supply two 1600 HP DEMU train sets?

(a) Bhutan (b) Myanmar (c) Bangladesh (d) Nepal

Q15) Apple is going to open its first retail store in which city of India?

(a) New Delhi (b) Bengaluru (c) Mumbai (d) Hyderabad

Q16) Which airlines has launched the world’s first-ever “zero-waste” flight?

(a) Qantas (b) Emirates (c) Ethiad (d) Spice Jet

Q17) Indian Army will observe year 2019 as which year?

(a) Year of Pulwama Martyrs (b) Year of Next of Kin (c) Year of Sacrifice (d) Year of Remembrance of Martyrs

Q18) India’s first UPI Bahi Khata for merchants has been started by which of the following?

(a) Paytm (b) BHIM (c) PhonePe (d) BharatPe

Page 43
STUDY IQ
bankiq.in MAY 2019

MCQ’S SOLUTION
Banking and Financial Awareness

Ans1-(b) 6 months

SEBI bars NSE from accessing the securities market for a period of six months
Securities and Exchange Board of India (SEBI) has barred National Stock Exchange (NSE) from accessing the securities market directly or
indirectly for a period of six months starting from April 30, 2019. It means NSE cannot launch a new derivative product for the next six months.
This has been done as NSE had not exercised the requisite due diligence while putting in place the TBT architecture. There were some
chances of fraud due to server co-location issue.
Tick-by-Tick (TBT) is a data feed, which provides information regarding every change in the order book on the NSE.

Ans2-(c) State Bank of India

State Bank of India links interest rates to RBI’s repo rate


India’s largest bank State Bank of India has moved to a new interest rate regime on large savings account deposits as well as short-term
loans. In March, SBI had announced that it will link its interest rate on savings account with balance above Rs. 1 lakh and short-term loans
like overdraft and cash credit facility to Reserve Bank of India’s repo rate, effective 1 May 2019.
The move assumes significance as the effective interest rate from May 1 on savings deposit above Rs 1 lakh will come down to 3.2% from 3.5%.
The interest rates on large SBI savings account deposits and interest rate on some short-term loans will automatically change as and
when RBI changes its repo rate. This will help in better transmission of RBI’s policy rates into the banking system.

Ans3-(b) India

India Was the Biggest Recipient of Funds from ADB in 2018


India was the biggest recipient of funds from the Asian Development Bank (ADB) in the year 2018.
The country received USD 3 billion in sovereign loans in 2018 from ADB.
This is the highest level of assistance since sovereign operations began in the country in 1986.
The multilateral funding institution is committed to grant sovereign loans in excess of USD 3 billion in 2019 as well.

Ans4-(b) Yes Bank

RBI slaps Rs 11 lakh fine on Yes Bank for violating money transfer norms
Yes Bank on May 3 said the Reserve Bank of India has imposed a penalty of Rs 11.25 lakh on it for violating money transfer norms.

Ans5-(d) All of the above

RBI slaps penalty on Vodafone m-pesa, PhonePe and 3 others


The Reserve Bank of India on Friday said it has imposed penalties on five prepaid payment instrument (PPI) issuers, including Vodafone
m-pesa and PhonePe, for violation of regulatory norms.
A penalty of ₹3.05 crore has been imposed on Vodafone m-pesa and ₹1 crore each on Mobile Payments, PhonePe, Private and GI Technology
Penalty on Western Union and MoneyGram has been imposed by the RBI under the provisions of the Payment and Settlement
Systems Act, 2007

Ans6-(b) Idai

Cyclone Idai: World Bank supports Mozambique, others with $545m


The World Bank (WB), on Friday disclosed a 545 million U.S. dollars donation to help people in Mozambique, Malawi, and Zimbabwe
that have been affected by Cyclone Idai.
This is in addition to nearly 150 million dollars in resources that the WB recently availed to the three countries.

Page 44
STUDY IQ
bankiq.in MAY 2019

“The World Bank is activating the International Development Association (IDA) Crisis Response Window (CRW) to provide up to 545
million dollars in total for the three affected countries.
“Together, total World Bank support to the three countries’ recovery reaches around 700 million dollars

Ans7-(c) $ 4.3 Billionl

India’s Foreign Exchange Reserves Rise By $4.3 Billion as on April 26: RBI
The country's foreign exchange reserves soared by $4.368 billion to $418.515 billion in the week to April 26, helped by the second
dollar-rupee swap auction, RBI data showed Friday.
In the swap auction conducted on April 23, the Reserve Bank had received bids worth $18.65 billion against $5 billion on offer. It
accepted just five bids worth $5 billion. In the previous week, the forex reserves had declined by $739.2 million to $414.147 billion.
In the reporting week, foreign currency assets, which are a major component of the overall reserves, rose by $4.387 billion to $390.421 billion.

Ans8-(b) Canada

Canada’s $10 bill adjudged as the best bank note for 2018 by IBNS
$10 bill of Canada has been adjudged as the best bank note for 2018 by the International Bank Note Society (IBNS).
This note of Bank of Canada features civil rights activist Viola Desmond (civil rights activist). Banknotes from 16 nations were in the race.
The IBNS Banknote of the Year award is an initiative to recognise an exceptional banknote issued every year

Ans9-(c) 5 Billion Dollar

ADB (Asian Development Bank) launched $5 Billion Healthy Oceans Action Plan at its 52nd Annual Meeting
Asian Development Bank (ADB) launched 5$ billion Action Plan for Healthy Oceans and Sustainable Blue Economies for the
Asia and Pacific region.
Main Aim: To achieve the Sustainable Development Goals (SDGs), inclusive of the SDG 14: Life below Water.
Among 10 rivers only 8 that transport 88% to 95% of plastics into the sea globally, 8 are in Asia and Pacific region.
It was launched at the 52nd Annual Meeting of ADB’s Board of Governors, Fiji.
It will expand financing and technical assistance for ocean health and marine economy projects to $5 billion from year 2019 to 2024.

Ans10-(d) Australia

Australian Central Bank prints 46 million piece of $50 note with spelling mistake
Reserve Bank of Australia (RBA) i.e the Central Bank of Australia has printed the $50 note with a spelling mistake where the word
“responsibility” is printed as “responsibilty”.
Around 400 million notes of $50 were printed and 46 million of them are in circulation. The total value of the notes in circulation is
around $2.3 billion.

Page 45
STUDY IQ
bankiq.in MAY 2019

Appointments and resignation

Ans1-(b) Rakesh Kumar Singh Bhadauria


Air Marshal RKS Bhadauria to Take Charge as Vice Chief of the Air Staff From May 1
Air Marshal Rakesh Kumar Singh Bhadauria will take over as the Vice Chief of Indian Air Forcefrom May 1, 2019 and would be looking
after all the air operations being carried out by the force.
He will succeed Air Marshal Anil Khosla who is superannuating on April 30, 2019.
Presently, Bhadauria is the Chief of IAF’s Bengaluru-based Training Command since August 1, 2018.

Ans2-(c) Dilip Kumar


Dilip Kumar Appointed as Officer on Special Duty in Office of Lokpal
Senior IAS officer Dilip Kumar has been appointed as the Officer on Special Duty (OSD) in the office of Lokpal on April 29, 2019. This is
the first official appointment of a bureaucrat in Lokpal.
Kumar is a 1995-batch IAS officer of Punjab cadre and is at present joint secretary at National Human Rights Commission.
He has been appointed for a period of six months on additional charge basis or till a new incumbent joins the post.
Chairperson of Lokpal -Justice Pinaki Chandra Ghose

Ans4-(b) B.V.P Rao

Archery Association of India (AAI) president Rao resigns after SC ruling


Archery Association of India (AAI) President B.V.P. Rao on Wednesday resigned after the Supreme Court’s ruling to set aside the
constitution of the National body of the sport as amended by Delhi High Court-appointed administrator S.Y. Quraishi.
The Supreme Court on Wednesday declared the amendments introduced by former chief election commissioner S.Y. Quraishi in the Constitution of the
Archery Association of India (AAI) as “null and void”, while holding the December 22, 2018 polls conducted to the body as “non est in law”.

Ans5-(b) Jayshree Vyas

BSE, formerly known as the Bombay Stock Exchange, has appointed its first independent woman director Jayshree Vyas. BSE already
has two non-executive women directors namely Usha Sangwan and Rajeshree Sabnavis on its Board.
Vyas is a professionally-qualified chartered accountant, working as the managing director of Shree Mahila Sewa Sahakari Bank,
Ahmedabad, since 1986. She helped in launching the first financial literacy programme in Sewa Bank in 2001.

Ans6-(a) Krishnamurthy Subramanian

Chief Economic Advisor Krishnamurthy Subramanian appointed as member of Advisory Council of 15th Finance Commission
The Chief Economic Advisor to Central government, Dr. Krishnamurthy Subramanian has been included as the member of the
Advisory Council of the Fifteenth Finance Commission.
Dr. Krishnamurthy Subramanian will be the twelfth Member of the Council.
The role and function of the Council is to advise the Commission on any issue or subject related to the Terms of Reference (ToR) of the
Commission.

Ans7-(d) Paytm

Paytm President Bhushan Patil Resigns


Bhushan Patil, president at Paytm, has quit the company. Patil, who joined Paytm after a five-year stint with Alibaba, had initially joined
the Noida-based company to look after its cross border commerce business. Patil joined Paytm in March 2016.

Page 46
STUDY IQ
bankiq.in MAY 2019

Ans8-(c) Senegal
Godavarthi Venkata Srinivas named India’s envoy to Senegal
ShriGodavarthiVenkataSrinivas (IFS:1993) was appointed as the next Ambassador of India to the Republic of Senegal.
Presently, he is Joint Secretary in the Ministry of External Affairs.
He succeeds Shri Rajeev Kumar, who is the High Commissioner of India to the Republic of Mozambique.
He is expected to take up the assignment shortly.

Ans9-(d) Reza Baqir


Pakistan appoints IMF official as new central bank governor
International Monetary Fund (IMF) economist Reza Baqir has been appointed as the Governor of State Bank of Pakistan.
State Bank of Pakistan is the central bank of Pakistan.
He will replace Tariq Bajwa, who was removed from his post.

Ans10-(d) Sanjiv Puri


Sanjiv Puri Appointed New Chairman and Managing Director of ITC
ITC Limited has appointed its MD Sanjiv Puri as the Chairman and Managing Director of the company with effect from May 13, 2019.
This decision was made following the demise of noted industry leader and erstwhile ITC Chairman Y C Deveshwar

Ans11-(a) KS Radhakrishnan

Former SC Judge KS Radhakrishnan appointed member of Commonwealth Secretariat Arbitral Tribunal


KS Radhakrishnan appointed as a member of Commonwealth Secretariat Arbitral Tribunal (CSAT) for a term of 4 year. Headquarter of
the CSAT is in London.
He had earlier served as a Supreme Court judge from November 2009 to May 2014. He has also served as the Chief Justice of the High
Courts of Gujarat and J&K

Ans12-(a) Tijjani Mohammad Bande

Nigerian Professor Muhammad-Bande appointed President Elect of UNGA


Tijjani Mohammad Bande, a professor from Nigeria, has been appointed as the new President of the United Nations General Assembly.
He will succeed incumbent María Fernanda Espinosa Garces after her tenure lapses in September 2019.
He served as the permanent representative of Nigeria to United Nations before his appointment as President of UNGA.
He will be the second Nigerian to be president of the UN General Assembly.
The first was Joseph Nanven Garba, who served as president between 1989 and 1990.

Ans13-(a) Balvinder Singh Nakai

Iffco elects Balvinder Singh Nakai as Chairman


Cooperative major IFFCO has elected Balvinder Singh Nakai as its Chairman and Dileep Sanghani as Vice Chairman.
Balvinder Singh Nakai is a farmer – co-operator – who has been deeply involved in providing strength to Indian cooperative movement.
Indian Farmers Fertiliser Cooperative Limited, also known as IFFCO is a Multi-state cooperative society engaged in the business of
manufacturing and marketing of fertilisers headquartered in New Delhi

Page 47
STUDY IQ
bankiq.in MAY 2019

Days and Dates

Ans1-(d) April 29
International Dance Day: April 29
The International Dance Day is observed globally on April 29 each year to encourage participation and education in dance through
events and festivals.
This day was created by the Dance Committee of the International Theatre Institute (ITI) which is the main partner for the performing
arts of UNESCO.
It marks the anniversary of the birth of Jean-Georges Noverre (1727–1810) who is known as the creator of modern ballet.
The day was created on the idea of jazz pianist and UNESCO Goodwill Ambassador Herbie Hancock.

Ans2(c)-30 April
Nation observes Ayushman Bharat Diwas on 30th April
As part of Gram Swaraj Abhiyan, Ayushman Bharat Diwas is celebrated across the country on April 30. For the first time, the day was
observed on 30th April 2018. During the event, information regarding Ayushman Bharat Scheme was provided to people during health
camps organised in rural areas.
On this day, gram sabhas were organized in different parts of the country to create awareness about the National Health Protection
Mission and verify and update details in the Socio-Economic and Caste Census database.

About Ayushman Bharat Scheme:


The Central Government on March 21, 2018, had approved the launch of a new Centrally Sponsored 'Ayushman Bharat -National Health
Protection Mission (AB- NHPM)'. Prime Minister Shri Narendra Modi has launched the scheme on April 14, 2018, at Bijapur, Chhattisgarh.
The scheme has the benefit cover of Rs. 5 lakh per family per year. The target beneficiaries of the proposed scheme will be more than 10
crore families belonging to poor and vulnerable population.

Ans3-(a) Uniting workers for Social and Economic Advancement

Labour Day or May Day celebrates labourers, working class and is an annual public holiday in many countries on May 1.
Labour Day has its origins in the labour union movement in the United States in the 19th century when the industrialists used to
exploit the labour class and made them work up to 15 hours a day. The workers rose against this exploitation and demanded paid leaves,
proper wages and breaks for the workforce. The eight-hour day labour movement advocated eight hours for work, eight hours for
recreation, and eight hours for rest.
The theme of Labour Day 2019 is "Uniting Workers for Social and Economic Advancement".
Labour Day or May Day honours the hard work of workers across the world and celebrate their achievements.

Ans4-(c) May 3

World Press Freedom Day was observed on May 3, 2019 which was proclaimed by the UN General Assembly in December 1993,
following the recommendation of UNESCO's General Conference.
The theme of World Press Freedom Day 2019 is, Media for Democracy: Journalism and Elections in Times of Disinformation.

Ans5-(c) May 3
May 3 observed as a National Space Day 2019
National Space Day is observed first Friday of May every year.
This year first Friday of May coincide with May 3, so science community celebrate this day to promote Science, Technology, Engineering
and Mathematics (STEM) education in young people and to inspire them to pursue a carrier in space-related jobs.

Page 48
STUDY IQ
bankiq.in MAY 2019

In 1997, Space Day was created by the Lockheed Martin Corporation as a one day event.
The main objective is to observe the many wonders of the unknown space that our planets float and also to encourage children to
have more interest in scientific study.

Ans6-(a) May 4
Coal Miners Day is observed on 4th May every year
Coal Miners Day is observed on 4th May every year to highlight the toughest profession and thepeople working in coal mines.
On this day several organisations, communities raises funds and awareness for other organisations in the coal mining area and workers.

Ans7-(a) May 5
World Laughter Day: 5 May
World Laughter Day takes place on the first Sunday of May of every year .
The first celebration was on May 10, 1998, in Mumbai, India, and was arranged by Dr. Madan Kataria, founder of the worldwide
Laughter Yoga movement.
Positive and powerful emotion that has all the ingredients required for individuals to change themselves and to change the world in a
peaceful and positive way.
The day is now celebrated worldwide.

Ans8-(c) May 7
World Asthma Day being observed
The World Asthma Day was observed across the globe on May 7, 2019, the first Tuesday of May 2019
According to the World Health Organisation, WHO there are 235 million people currently suffering from asthma around the world.
Asthma is one of the major non-communicable diseases and it is also the most common chronic disease in children. Regular exercise
can help reduce asthma symptoms.
The first World Asthma Day, in 1998, was celebrated in more than 35 countries in conjunction with the first World Asthma Meeting
held in Barcelona, Spain.

Ans9-(c) May 11
World Migratory Bird Day 2019: May 11
The World Migratory Bird Day (WMBD) is celebrated twice in a year since it started in 2006.
Firstly it is held on Second Saturday of May and again on Second Saturday of October.
In 2019, the WMBBD is being held on May 11 and October 12.
The day is observed to spread awareness for the need to conserve migratory birds and their habitats
The theme for 2019 WMBD is “Protect Birds: Be the Solution to Plastic Pollution.”

Ans10-(a) May 11
National Technology Day: 11th May
National Technology Day is observed every year on 11th of May in India, acting as a reminder of the anniversary of Shakti.
Shakti is the Pokhran nuclear test which was held on the 11th of May, 1998. The first nuclear test Pokhran with a codenamed as
‘Smiling Buddha’ was carried out in May 1974.

Ans11-(b) May 7

World Athletics Day (2019): History and Purposes


The World Athletics Day is organized by the IAAF (International Amateur Athletic Federation). World Athletics Day was
celebrated for the first time in 1996.
In the year 2019; the World Athletics Day is celebrated on 7 May.
On which day the Athletics Day will be observed is decided by the IAAF. In the year 2006 the participants of this day were between the
age of 7 to 15 years but later on this age gap made to 13 to 17 years.

Page 49
STUDY IQ
bankiq.in MAY 2019

Defence and security

Ans1-(d) South Korea


INS Kolkata and INS Shakti Reaches South Korea to Participate in ADMM-Plus Maritime Security Exercise.
The Indian naval ships INS Kolkata and INS Shakti reached Busan in South Korea on April 28 to participate in the three days ADMM-Plus
Maritime Security Field Training Exercise (FTX). The exercise is a part of deployment of the Eastern Fleet to the South China Sea.
The two ships recently completed the international fleet review in Qingdao, China, as part of the celebrations of the 70th founding
anniversary of the Chinese People’s Liberation Army Navy.
The ADMM-Plus (Asean Defence Ministers’ Meeting-Plus) is a platform for the ASEAN states and its dialogue partners to strengthen
security and defence cooperation for peace, stability and development in the region.
The ADMM-Plus countries include ten ASEAN Member States and eight dialogue countries, namely Australia, China, India, Japan, New
Zealand, South Korea, Russia and the United States.

Ans2-(b) Defence Cyber Agency


Defence Ministry set to have three new tri-service agencies
Ministry of Defence (MoD) is expecting to raise a Defence Cyber Agency (DCA) by May to tackle non-civilian cyber issues arise from
China and Pakistan, including safeguarding critical infrastructure. Rear Admiral Mohit Gupta, Senior Navy officer will be the first head of
this agency, headquartered in New Delhi.
As per the clearance given by Prime Minister Narendra Modi in 2018, two more agencies are under construction namely, Defence
Space Agency and Special Forces Agency.
The Special Forces agency would be headed by a Major General rank officer of the Army and is likely to come up at Agra.

Ans3-(c) France
Indo-French Joint Naval Exercise “Varuna-19” Commences off Goa Coast
The 17th edition of the Indo-French joint naval exercise, “Varuna-19”, began on May 1 off the Goa coast.
The exercise will be conducted in two phases.
The first part of the exercise, Varuna 19.1, is the harbour phase that will be held off the Goa coast from May 1-10, 2019.
The second part, Varuna 19.2, is the sea phase which is scheduled to be held at the end of May 2019 in Djibouti.

Ans4-(a) INS Vela


Indian Navy Launches 4th Scorpene Class Submarine INS Vela
The fourth of Indian Navy’s stealth Scorpene class Submarines of Project 75, INS Vela was launched at the Kanhoji Angre Wet Basin of
Mazagon Dock Limited at Mumbai in Maharashtra.
The submarine has been named ‘Vela’ after an earlier submarine, which was the lead submarine, of the erstwhile Vela class, the second
batch of the Foxtrot class submarines acquired from erstwhile USSR.

Page 50
STUDY IQ
bankiq.in MAY 2019

Honours and Awards


Ans1-(c) Alfred Brownell
Alfred Brownell was awarded the Goldman Environmental Prize
Alfred Brownell, an environmental lawyer, and activist was awarded the Goldman Environmental Prize. He was awarded for exposing
alleged abuse by the palm oil company Golden Veroleum Liberia and helping to prevent it from converting about 50 km2 of forest that
is home to elephants, pygmy hippopotamuses, and chimpanzees.

Other Awardees:
Linda Garcia of Vancouver, Washington, who rallied local communities to successfully prevent the construction of North America’s
largest oil terminal.
Ana Colovic Lesoska of North Macedonia, whose seven-year campaign helped stop hydroelectric projects from being built in the
country’s largest national park.
Bayarjargal Agvaantseren of Mongolia, who led the fight to create the Tost Tosonbumba Nature Reserve.
Jacqueline Evans of the Cook Islands, whose work led to the conservation and sustainable management of all of the Cook Islands’ ocean
territory and creation of 15 marine protected areas.
Alberto Curamil of Chile, a jailed indigenous activist who had protested several hydroelectric projects in the country.

Ans2-(c) Israel
Israel honours 1971 war hero Lt Gen JFR Jacob with a plaque on Ammunition Hill Wall of Honour
Israel on Tuesday honoured Lt. Gen. (retd) Jack Farj Rafael (JFR) Jacob, who negotiated the historic surrender of Pakistani troops in
Dhaka after the 1971 Bangladesh's liberation war, with a plaque on the Wall of Honour at the Ammunition Hill.

Ans3-(a) A S Kiran
Former ISRO Chairman awarded France's Highest Civilian Honour
Former ISRO chairman A S Kiran Kumar was on May 2, 2019 conferred with France's highest civilian honour, Chevalier de l'Ordre
national de la Legion d'Honneur, for his contribution to India-France space cooperation.
Kumar was conferred with the prestigious honour by France’s Ambassador to India Alexandre Ziegler on behalf of the French President.

Ans4-(c) Tod D Wolters


US Air Force General Tod D. Wolters declared as the top military officer of the 29-nation NATO military alliance.
Tod D. Wolters, the former pilot, who has served in Afghanistan and Iraq, has been declared as the Supreme Allied Commander Europe
at a ceremony at NATO‘s (The North Atlantic Treaty Organization) military headquarters. This is one of the most challenging and
extremely important military positions in the world.
US Air Force General Tod D. Wolters who is supposed to run for a tenure of 2 to 3 years, was preceded by U.S. Army General, Curtis M. Scaparrotti.

Ans5-(a) Nileena M S
Article on coalgate wins ACJ Award for Investigative Journalism
The article titled Coalgate 2.0' authored by Nileena M S and published in The Caravan magazine in March last year has won the ACJ
Award for Investigative Journalism, 2018.
The final jury comprising Gopalkrishna Gandhi, Nilita Vachani and Dr A R Venkatachalapathy Friday unanimously decided on Nileena's
work as the winner, says an ACJ release.

Page 51
STUDY IQ
bankiq.in MAY 2019

Ans6-(a) Devi Prasad Shetty


PC Chandra Purashkaar 2019 conferred to Padma Bhushan Dr. Devi Prasad Shetty.
India’s most reputed cardiac surgeon and humanitarian, Dr. Devi Prasad Shetty was awarded 27th P.C. Chandra Puraskaar, the annual
national award from the P.C. Chandra Group at Biswa Bangla Convention Centre.
General Secretary of Ramakrishna Mission, Belur Math, Swami Suviranandaji Maharaj graced the occasion as the Chief Guest and gave away the award.
He is also known as ‘The Man with Divine Hands’.
In 2003, he has won Padma Shri and Padma Bhushan for medicine in 2013.

Ans7-(b) Jitender Kumar, Shikha Garg


Two Indians among 115 UN personnel staff honoured for sacrifice in line of duty
An Indian police officer Jitender Kumar deployed to the UN mission in Congo and a consultant Shikha Garg with the UN development
programme, killed in the Ethiopian Airlines plane crash, are among 115 UN peacekeepers and staff honoured by the global organisation
for their sacrifice. They lost their lives in the service of peace between January 2018 and March 2019
UN Secretary General Antonio Guterres, along with top UN officials and Peacekeeping personnel paid tribute to 115 peacekeepers
from 43 different nations, who sacrificed their lives in the line of duty at the solemn ceremony yesterday.
UN Secretary General Antonio Guterres, along with top UN officials and Peacekeeping personnel paid tribute to 115 peacekeepers
from 43 different nations, who sacrificed their lives in the line of duty at the solemn ceremony yesterday.

Ans8-(b) G D Robert Govender


Indian-origin journalist conferred with V K Krishna Menon award
G D ‘Robert’ Govender has been awarded the 2019 V K Krishna Menon award posthumously, for his outstanding contribution as a
pioneer of decolonised journalism.
He is an Indian Origin Journalist. He was awarded in UK.

Ans9-(a) Chhaya Sharma


Officer Who Led Nirbhaya Case Receives Award for Courage and Leadership
IPS officer Chhaya Sharma, who led the investigation team of the infamous Nirbhaya gang-rape and murder case, was honoured with
the 2019 McCain Institute Award for Courage and Leadership on Friday.
"A resolute guardian of human rights, Chhaya has led teams in detection and investigation of sensitive criminal cases and protection of
human rights during her career spanning over 19 years. Sharma's policing has consistently reflected victim-centric approach through her
numerous investigations of serious crimes, particularly against women and children," said McCain Institute in a statement.

Ans10-(b) G D Robert Govender


Indian-origin journalist conferred with V K Krishna Menon award.
G D ‘Robert’ Govender has been awarded the 2019 V K Krishna Menon award posthumously, for his outstanding contribution as a
pioneer of decolonised journalism.
He is an Indian Origin Journalist. He was awarded in UK.

Page 52
STUDY IQ
bankiq.in MAY 2019

India and the World

Ans1-(a) Netherlands
Largest Foreign Direct Investor In India
The Netherlands has emerged as the third largest foreign direct investor in India during 2017-18, with investments pegged at about
$2.67 billion across sectors.
The Netherlands was also the second largest destination for foreign investment by Indian companies, after Singapore, with
investments worth $12.8 billion in 2017.

Ans2-(c) 29
India extends retaliatory tariff deadline on US products till May 16.
The Government has extended its deadline to impose retaliatory import duties on 29 products from the United States (US), including
almond, walnut and pulses, till May 16.
A notification of the Finance Ministry has said that the implementation of increased customs duty on specified imports, originating in
the US, has been postponed from May 2 to May 16 this year.
These deadlines were extended several times since June 2018, when India decided to impose these duties in retaliation to a move by
the US to impose high customs duties on certain steel and aluminium products.

Ans3-(C) Canberra

India-Australia Joint Working Group on Counter-Terrorism


The 11th meeting of the joint working group on counter-terrorism between India and Australia held at Canberra, Australia on May 2, 2019.
India and Australia have agreed to further deepen counter-terrorism co-operation through regular exchange of information and
sharing of best practices on countering extremism and radicalization.

Ans4-(b) INS Vishal


India in talks with Uk for INS Vishal
The United Kingdom is in talks with the Indian government on building a new state-of-the-art aircraft carrier along the lines of Britain’s
HMS Queen Elizabeth as part of the ongoing Make in India negotiations.
The talks are underway for the Indian Navy to buy detailed plans for the 65,000-ton British warship to build a so-called copycat
supercarrier to be named INS Vishal in 2022.
If a deal can be agreed, the new warship would be built in India but UK companies could supply many of the parts.
INS Vishal:
INS Vishal, meaning 'giant' in Sanskrit, also known as Indigenous Aircraft Carrier 2 (IAC-2) is a planned aircraft carrier to be built by
Cochin Shipyard Limited for the Indian Navy.

Page 53
STUDY IQ
bankiq.in MAY 2019

Industry

Ans1-(b) IBM
Vodafone Idea collaborates with IBM and signs a multi-million dollar IT deal
Vodafone Idea Ltd, India’s largest telecom operator notified a 5-year multi-million-dollar IT outsourcing deal with IBM (International
Business Machines Corporation).
The collaboration will contribute to Vodafone Idea’s merger synergy objectives by decreasing its IT related costs.
This partnership will offer Vodafone Idea with a hybrid cloud based digital platform to enable engagement with its 387 million
subscribers (as of 31 December, 2018), thus boosting business efficiency, scale and easiness of its business processes.
Moreover, IBM’s analytics, AI (Artificial Intelligence), IoT (Internet of Things) and security capabilities will boost Vodafone Idea’s progression.

Ans2-(c) Assam
ONGC gets green nod for Rs 240cr project in Assam
State-owned Oil and Natural Gas Corporation (ONGC) has received the green nod to drill 6 development wells in Assamat an
estimated cost of Rs 240 crore.
The Union Environment Ministry has given the environment clearance for six development wells in five mining lease blocks in Jorhat
and Golghat districts.
The company had sought permission for drilling 12 development wells but it received clearance for six wells at present. The total
mining lease area of Jorhat and Golaghat districts is 32.116 sq km and 120.5 sq km, respectively

Infrastructure

Ans1-(a) Bengaluru
LIC HFL launches Udyam Centre in Bengaluru
LIC Housing Finance (LIC HFL) has launched ‘Udyam’, a skilling centre in Bengaluru. It was launched in association with Lok Bharti
Education Society, the implementing partner.
The Centres of Excellence will provide training to marginalised youth in the BFSI, retail, and IT/ ITES sectors

Page 54
STUDY IQ
bankiq.in MAY 2019

International Affairs

Ans1-(d) US
US to withdraw from the U.N. Arms Trade Treaty.
US President Donald Trump stated that his administration is withdrawing the U.S. signature from the U.N. Arms Trade Treaty, in
response to concerns from gun rights activists that it could impinge on Americans’ right to bear arms. Former US President Barack
Obama signed the treaty in 2013 and sent the treaty to the Senate for ratification.
The Trump administration has rejected or withdrawn from a number of international agreements signed by his predecessors, including
the Iran nuclear deal.

Ans2-(c) Pakistan
Pakistan has issued an official order to freeze the assets of and impose a travel ban on terror outfit Jaish-e-Mohammed (JeM) chief
Masood Azhar after the United Nations declared him a “global terrorist.
Azhar is also banned from selling or purchasing arms and ammunition.
The UN sanctions committee on the Islamic State and Al-Qaeda on Wednesday announced the designation of Azhar over his ties to Al-Qaeda.

Ans3-(a) North Macedonia


Stevo Pendarovski wins North Macedonia's presidential election
Stevo Pendarovski, supported by the ruling Social Democrats (SDSM), has won the North Macedonian presidency in a run-off vote.
The 56-year-old garnered 51.66 percent of the vote in North Macedonia on Sunday, beating nationalist VMRO's Gordana
Siljanovska-Davkova which got 44.73 percent. The remaining 3.61 percent of the votes were void.

Ans4-(c) USA
US Air Force Successfully Tests Laser System ‘SHiELD’ to Shoot Down Missiles
The United States Air Force successfully conducted the test of a laser weapons system called the Self-Protect High Energy Laser
Demonstrator (SHiELD) in April 2019.
SHiELD is an Advanced Technology Demonstration (ATD) Program which is capable of shooting down multiple missiles in flight.
It would be mounted on aircraft to demonstrate self-defense of aircraft and protect the fighter jets against surface-to-air (SAM) and
air-to-air (AAM) missiles attack.

Ans5-(d) $6 billion
Pakistan reaches agreement with IMF, to get $6 billion over 3 years.
Pakistan on Sunday reached an agreement with the IMF on a bailout package under which the cash-strapped country will receive $ 6
billion over three years, according to a top official

Ans6-(c) Japan
Japan Tests World’s Fastest Bullet Train With Speed of 400 kmph
Japan began the testing of the world’s fastest bullet train called ALFA-X on May 10, 2019 and this testing will continue for a period
spanning three years.
The latest version of Shinkansen train is capable of reaching speeds as much as 400 kilometers per hour (249 miles per hour).
The train is scheduled to go into service in 2030.
When in operation, ALFA-X will operate at 360 kph (224 mph).
It will then surpass China’s Fuxing train which is currently the fastest train running at the speed of 350 kph

Ans7-(c) Panama
Laurentino Cortizo Wins Presidential Elections of Panama
Laurentino Cortizo, member of the Democratic Revolutionary Party, has won the presidential elections of Panama with 33.27% of the
vote in support.
The 66-year-old will take office on July 1, 2019. Page 55
STUDY IQ
bankiq.in MAY 2019

Polity and Governance

Ans1-(d) 50%
SC rejects review petition for 50% VVPAT verification
The Supreme Court on May 7, 2019 rejected the review petition filed by 21 opposition parties seeking 50 percent verification of Voter
Verified Paper Audit Trail (VVPAT) during counting of votes in Lok Sabha Elections 2019.
A bench headed by Chief Justice Ranjan Gogoi said, "We are not inclined to review our order".
Earlier on April 8, 2019, the apex Court had directed the Election Commission of India (ECI) to increase the random checking of VVPAT
slips from 1 EVM to 5 EVMs in each assembly of a parliamentary constituency during the ongoing Indian General Elections 2019.

Ans2-(d) Maharashtra

EC allows poll code relaxation for Maha drought relief works


The Election Commission of India has permitted relaxation in the model code of conduct for the Maharashtra government to
undertake drought relief works.
The relaxation was made after chief minister Devendra Fadnavis wrote to the EC on April 30 on the issue. Polling for all the 48 Lok
Sabha seats in the state got over on April 29.

Reports and Indices

Ans1-(c) 2050
Drug-resistant diseases could kill 10 million a year by 2050
Drug-resistant diseases could cause 10 million deaths each year by 2050, warned the UN Ad Hoc Interagency Coordinating Group on
Antimicrobial Resistance in a report released on Monday.
It added that by 2030, antimicrobial resistance could force up to 24 million people into extreme poverty.
-“Currently, at least 7,00,000 people die each year due to drug-resistant diseases, including 2,30,000 people who die from
multidrug-resistant tuberculosis,” said the report.

Ans2-(b) Fourth

India 4th largest military spender; Top – US- SIPRI Report


World's military expenditure increased by 2.6 per cent, reaching $1822 billion in 2018 with India along with United States, China, Saudi
Arabia and France contributing 60 per cent of the amount, according to a latest report from the Stockholm International Peace Research
Institute (SIPRI).
India was the world’s fourth largest military spender in the year with spending of $66.5 billion on military. There was a rise of 3.1% in
the spending.
The country with the maximum military spending as per SIPRI is the United States, followed by China on second and Saudi Arabia on
the third.

Ans3-(d) 7.3%
India Ratings predicts India’s GDP growth rate at 7.3% for FY20
India Ratings and Research has predicted India’s GDP growth rate prediction for 2019-20 at 7.3%. This is less than its earlier prediction
of 7.5%.

Page 56
STUDY IQ
bankiq.in MAY 2019

Ans4-(d) 40%
Indian cyber insurance market grows 40% with 350 cyber insurance policies bought in 2018 against 250 in 2017
A report titled ‘Cyber Insurance in India- Mitigating Risks amid Changing Regulations and Uncertainties‘ by Data Security Council of
India (DSCI) stated that cyber insurance market is gradually gaining momentum in India as cyber-theft has increased substantially to
become one of the most prominent threats to the corporates’ operations.
The report broadly offers market insights on the stances in cyber insurance with its prime objective to increase awareness and help
evaluate a cyber insurance policy effectively.
Data from the report showed that between 2016 and 2018, India ranked as the 2nd most affected country from cyber-threats. 350 cyber
insurance policies were bought by Indian corporates in 2018 as compared to 250 in 2017, which marks a 40% hike in the sale of these policies.

Ans5-(c) London

London top choice for Indian investors with record investments in 2018: Report
London emerged as the top choice for Indian investors ahead of other cities such as Dubai and Singapore, with investments by Indian
companies in the British capital reaching an all-time high last year, according to a new analysis.
The UK emerged as the top most country to attract Indian FDI with 52 projects, ahead of the US (51) and the UAE (32), in 2018, said
London & Partners (L&P), the Mayor of London's promotional agency, in the new analysis.
Indian companies choosing to invest and expand in London reached an all-time high last year, with 32 investment projects – a giant leap
from the previous year, it said.
"Foreign Direct Investment (FDI) from India into London increased by 255 per cent from 2017 to 2018.

Sports

Ans1-(c) Valtteri Bottas


Valtteri Bottas wins 2019 Azerbaijan Grand Prix
Mercedes racing driver Valtteri Bottas won the 2019 Azerbaijan Grand Prix on April 28, 2019.
With this, the Finnish player claimed the Formula One world championship lead from teammate Lewis Hamilton, who was second.
Ferrari’s Sebastian Vettel came in third place while Red Bull’s Max Verstappen stood fourth followed by Ferrari’s Charles Leclerc came in
fifth position.

Ans2-(a) Petra Kvitova

Petra Kvitova Clinches Stuttgart’s Porsche Grand Prix 2019


World number three Petra Kvitova beat Estonian Anett Kontaveit 6-3 7-6(2) to win Stuttgart’s Porsche Grand Prix 2019 held in Stuttgart,
Germany from April 22-28, 2019.
This was the second title of the year for the Czech based Kvitova while the 27th of her career.
Mona Barthel / Anna-Lena Friedsam (Germany) defeated Anastasia Pavlyuchenkova (Russia) / Lucie Šafářová (Czech) to clinch the
doubles title at the tournament.

Ans3-(d) Bajrang Punia

Bajrang Punia to become the first Indian wrestler to fight at Madison


Bajrang Punia is set to become the first wrestler from India to fight at the iconic Madison Square Garden in New York.
The 25-year-old, who won the gold medal in the 65-kg category at the ongoing Asian Wrestling Championships earlier this week, is the
first Indian wrestler to be invited by the American governing body.

Page 57
STUDY IQ
bankiq.in MAY 2019

Bajrang Punia will take part in the “Grapple at the Garden — Beat the Streets” fight night, scheduled to be held on May 6. He is
currently the World No.1 in the 65-kg weight category.

Ans4-(a) Raheem Sterling


Raheem Sterling has been named the 2019 footballer of the year by the Football Writers' Association (FWA).
The Manchester City and England forward topped the poll of the 400-strong FWA membership as a clear winner, with 62 percent of the votes
It was more than 100 votes ahead of Liverpool defender Virgil Van Dijk.

Ans5-(a) G Sathiyan
G Sathiyan becomes first Indian to get into top 25 ITTF rankings
India's G Sathiyan on Monday scaled new heights in his rapidly rising career by becoming the first Indian to break into the top-25 of the
latest world ranking list released by International Table Tennis Federation.
Sathiyan raise four spots to be 24th in the ITTF rankings, following his impressive sixth place finish in the Asia Cup in Yokohama earlier
this month to secure a place in the World Cup in Chengdu from October 25 to 27. Last week, he was the sole Indian to reach the round of
32 in the World Championships in Hungary.

Ans6-(a) Stephanie Frappart

First female Ligue 1 referee takes charge of match


Stephanie Frappart became the first woman to referee a French Ligue 1 match. The 35-year-old Frappart took charge as Amiens hosted
Strasbourg, where she was called into action numerous times, handing out yellow cards to the likes of Strasbourg’s Ibrahima Sissoko and
Ludovic Ajorque.
Frappart is not the first woman to officiate a top-flight men’s game in Europe’s top leagues. Germany’s Bibiana Steinhaus became the
first woman to officiate a Bundesliga match in 2017

Ans7-(a) Barcelona

Barcelona Clinch Back-to-Back La Liga Titles With Win Over Levante


Barcelona officially sealed another La Liga title at Camp Nou on Saturday night, with Lionel Messi scoring the only goal in a 1-0 win over Levante.
After Atletico Madrid beat Real Valladolid earlier in the day, Barça knew that a home win was required to mathematically secure
consecutive league crowns, ahead of their Champions League semi-final with Liverpool on Wednesday

Ans8-(a) Kumar Sangakkara


Kumar Sangakkara named as first non-British President of MCC
Former Sri Lanka captain Kumar Sangakkara appointed as the first non-British president of the Marylebone Cricket Club.
Sangakkara will be the first overseas MCC president in the organization's 233-year history.

Ans9-(c) Apurvi Chandela


Apurvi Chandela attains world number one spot in women's 10 m air rifle event
Indian shooting ace Apurvi Chandelaattained the world number one position in the women's 10 metres air rifle event.
Anjum Moudgil attained number two after consistent performances in recent years.
The rifle ace from Jaipur is among the five Indian shooters to have already secured 2020 Olympics quotas for the country.
Moudgil attained the world number two rank in 10 metres Air Rifle after her mixed team gold at the ISSF World Cup in Beijing.
Manu Bhaker is the world number 10 in 25 metres pistol women category.

Page 58
STUDY IQ
bankiq.in MAY 2019

Ans10-(b) 2024
India to bid 2024 edition of the biennial World Team Championships
Table Tennis Federation of India (TTFI) has decided to bid for the 2024 edition of the biennial World Team Championships.
The last time India hosted a World Championships was way back in 1987.
In the 2024 edition, 32 teams will take part. Team Championships take place on even years and Individual Championships on odd years.
The last big ITTF event that was staged in India was in 2012 when Hyderabad hosted the World Junior Championships.

Ans11-(b) Bajrang Punia


World number one Bajrang Punia on May 2, 2019 won the gold medal after defeating Viktor Rassadin 13-8 in the finals of men's 65kg
freestyle in the 2019 Ali Aliyev Wrestling Tournament in Kaspiisk, Russia. This is his second title in two weeks. Earlier on April 23, Bajrang
won gold in the same category, beating Kazakhstan’s Sayatbek Okassov 12-7 in the final of the Asian Wrestling Championships in Xi'an, China.

Ans12-(b) David Warner


Mumbai Indians Defeats Chennai Super Kings to Win Record 4th IPL title
Mumbai Indians defeated the Chennai Super Kings by 1 run in the finals to win their record fourth title in Indian Premier League (IPL)
on May 12, 2019 at Rajiv Gandhi International Cricket Stadium, Hyderabad
The 2019 season of the IPL was the twelfth season since the professional Twenty20 cricket league was established by the Board of
Control for Cricket in India (BCCI) in 2007.
Most successful wicket-keeper – Mahendra Singh Dhoni became the most successful wicket-keeper in the history of IPL by taking his
dismissals tally to 132, surpassing his India teammate Dinesh Karthik (131).
Orange Cap (Most Runs)- David Warner (SRH) for being the leading run-scorer of the tournament with 692 runs.
Purple Cap (Most Wickets)- Imran Tahir (CSK) for finishing as the leading wicket-taker of the tournament with 26 wickets.

Ans13-(b) Sana Mir


Sana Mir Becomes Highest Wicket-Taking Spinner In ODI
Pakistan off-spinner Sana Mir has become the most successful women’s ODI spinner in history after she took her 147th wicket, highest
ever by any spinner in One-day International (ODI) cricket.
Currently, Mir is placed on the third position in the ICC’s ODI bowling ranking, which is topped by Indian pacer Jhulan Goswami.

Ans14-(d) Harbhajan Singh


Harbhajan Singh becomes third Indian to take 150 wickets in IPL
Chennai Super Kings (CSK) off-spinner Harbhajan Singh became the third Indian to claim 150 wickets in the Indian Premier League (IPL).
The bowler achieved the feat in the second qualifier against Delhi Capitals at Y.S. Rajasekhara Reddy ACA-VDCA Cricket Stadium in Visakhapatnam.
Delhi’s Amit Mishra and Kolkata Knight Riders’ Piyush Chawla are the other two Indians who have taken 150 wickets in the IPL.

Ans15-(d) Igor Stimac


Igor Stimac set to be appointed Indian football team coach
Croatia’s World Cupper and former manager Igor Stimac has been appointed as Indian men’s team coach after the All India Football
Federation’s (AIFF) technical committee recommended him for the post. He succeeds Stephen Constantine.

Page 59
STUDY IQ
bankiq.in MAY 2019

Miscellaneous

Ans1-(d) Kyrgyzstan

Nirmala Sitharaman Participates in SCO Defence Ministers’ Meet in Bishkek


The Defence Minister Smt. Nirmala Sitharaman is on a three days visit to Bishkek in Kyrgyzstan to attend the Shanghai Cooperation
Organisation (SCO) Defence Ministers’ Meet.
The SCO meeting is being held in the run-up to the SCO summit, which is expected to take place in June 2019.
India participated in the SCO Defence Ministers meeting for the first time in 2018.

Ans2-(c) Paytm

Paytm Launches Recurring Payment Options for start-ups.


Noida-based e-commerce payment system and digital wallet company, Paytm launched a new option of recurring payments for its
merchants.
Recently it also started its own subscription business at an annual fee of Rs 750 where consumers pay at regular intervals.
Consumers can opt for recurring payments from Paytm wallet, also from their bank accounts, debit cards, credit cards, net banking
and more.

Ans3-(b) Bharti AXA

Bharti AXA General Insurance, the 1st company to sell 2-wheeler policies via WhatsApp, by partnering with Wishfin.
Bharti AXA General Insurance, the first company in the domestic non-life insurance industry has collaborated with web aggregator,
Wishfin Insurance to sell 2-wheeler policies via social media messaging platform, WhatsApp.
Bharti AXA General Insurance is a joint partnership between Bharti Enterprises and AXA, global insurance firm. It will sell the policies on
Wishfin’s insurance arm, Wishpolicy’s

Ans4-(a) Madras

Former Chief Justice of Madras, Kerala High Courts B Subhashan Reddy Passes Away
B Subhashan Reddy, the former Chief Justice of Madras and Kerala High Courts passed away due to ill-health.
He was 76.
He had also served as the judge of the Andhra Pradesh High Court and Chairman of State Human Rights Commission and Lok Ayukta.

Ans5-(d) Green

ISRO to launch radar imaging satellite ‘RISAT 2BR1’


India is planning to launch its latest radar imaging satellite RISAT 2BR1 towards the end of May 2019 on one of the variants of the Polar
Satellite Launch Vehicle (PSLV) rocket.
The satellite, RISAT 2BR1, is expected to be launched aboard one of the reusable PSLV rockets on May 22, as per officials of the Indian
Space Research Organisation (ISRO).
The rocket that would carry the radar imaging satellite is designated as PSLV-C46 as per ISRO's numbering system.

Ans6-(d) RISAT 2BR1

ISRO to launch radar imaging satellite ‘RISAT 2BR1’


India is planning to launch its latest radar imaging satellite RISAT 2BR1 towards the end of May 2019 on one of the variants of the Polar
Satellite Launch Vehicle (PSLV) rocket.
The satellite, RISAT 2BR1, is expected to be launched aboard one of the reusable PSLV rockets on May 22, as per officials of the Indian
Space Research Organisation (ISRO).
The rocket that would carry the radar imaging satellite is designated as PSLV-C46 as per ISRO's numbering system.

Page 60
STUDY IQ
bankiq.in MAY 2019

About RISAT
The RISAT, which was first deployed in orbit on April 20, 2009 as the RISAT-2, uses synthetic aperture radars (SAR) to provide Indian forces
with all-weather surveillance and observation, which are crucial to notice any potential threat or malicious activity around the nation's

Ans7-(c) Mangaluru

Mangaluru police gets an all-women patrol unit Rani Abbakka Force.


Mangaluru City Police launched an all-women police patrol unit ‘Rani Abbakka Force’.
The force will cover the malls, beaches, educational institutions, places of religious importance, and the public transport system and
handle issues related to eve-teasing and chain-snatching, among others.
All-women police patrol force named after the 16th-century warrior of coastal Karnataka, Queen Abbakka of Ullal near Mangaluru, who
fought the Portuguese in the 16th century.

Ans8-(b) Qatar

16th Ministerial Meeting of Asia Cooperation Dialogue Held in Qatar


The 16th Ministerial Meeting of Asia Cooperation Dialogue (ACD) was held in Doha, Qatar.
Gen. (Dr) V.K. Singh (Retd), Minister of State for External Affairs led the Indian delegation. The theme of the meeting was “Partners in Progress”.

Ans9-(c) Russia

India to purchase 10 Kamov-31 choppers from Russia for 3,600 crores.


Defence Ministry Nirmala Sitharaman approved Indian Navy's proposal to acquire 10 Kamov-31 choppers from Russia at a cost of
around Rs. 3,600 crore to strengthen its capability against aerial threats to its aircraft carriers and large warships.
They cleared the Rs. 3,600 crore proposal for buying around 10 Kamov-31 Airborne Early Warning and Control choppers for the aircraft
carrier operations and deployment on future warships.
Indian Navy already has a fleet of 12 of these Kamov-31 choppers which sanitise the air space around the aircraft carriers and destroyers
operating in open seas.

Ans10-(a) Aditya-L1

ISRO planning to send a probe to study sun in 2020


The Indian Space Research Organization is planning to send a probe to study the sun early in 2020.
The information was shared by ISRO chairman Dr K Sivan, who said that currently scientists are exploring possibilities to study more
about Sun and the probe named Aditya-L1 will be sent to observe the solar corona, the outer layers of the Sun.
The satellite would be in a halo orbit around the L1 (Lagrangian point 1) of the Sun-Earth system so that it has the advantage of
continuously viewing the Sun without any occultation or eclipses. The L1 is 1.5 million kms from the Earth.

Ans11-(b) SIPRI

GReVD: SIPRI launches Global Registry of Violent Deaths


Stockholm International peace research institute (SIPRI) has launched a new initiative named Global Registry of Violent Deaths (GReVD)
to establish the annual number of violent deaths worldwide.
GReVD will count deaths caused by all forms of violence and display these in an open-source database and enable monitoring of
progress on the world’s commitment to ‘significantly reduce all forms of violence and related death rates everywhere’ by 2030 as set out
in the Sustainable Development Goals (SDGs)—Goal 16.
The database will enable policymakers and researchers and the general public to track trends in violence at global, regional, national,
provincial, city and municipal levels.

Ans12-(b) Shahid Afridi

Pakistan’s former captain and star all-rounder Shahid Afridi recently released his autobiography, titled ‘Game Changer’.
The book, co-authored by Afridi and journalist Wajahat S Khan, details some interesting and exciting information from the cricketer’s life

Page 61
STUDY IQ
bankiq.in MAY 2019

Ans13-(b) France
India to attend Group of 7 industrialized nations meet
The Group of 7 (G7) industrialized nations meet is organized by France in Biarritz in August 2019.
As India has been invited by France, India will be attending the meet along with Australia, Chile, and South Africa, in a sign of the
recognition of India’s growing economic heft in the international arena. The invitation by France clearly reflects the strong strategic connect
between India and France.
The G-7 groups constitute of the US, Britain, France, Italy, Canada, Japan, and Germany.
The French invite to India is not the first time India will be at this G-7 global high table. Then Prime Minister Manmohan Singh had
attended the G-8 +5 meet in Gleneagles in Scotland in 2005 at the invitation of then British Prime Minister Tony Blair.

Ans14-(d) Nepal

Konkan Railways Signs Contract Agreement With Nepal To Supply DEMU Train Sets
Konkan Railways Corporation Ltd has signed a contract agreement with Nepal's Department of Railways to supply two 1600 HP DEMU
train sets.
These trains will be used to operationalise railway link between Jayanagar in India and Kurtha in Nepal. The train sets will be
manufactured by the Integrated Coach Factory, Chennai.

Ans15-(c) Mumbai

Apple to open its first retail store in India in Mumbai.


Apple Inc has finalized a short list of locations for its first retail store in India. It will open its first retail store in India in Mumbai.
However, the final place is not finalized yet.

Ans16-(a) Qantas

Australian Airline Launches World’s First-Ever ‘Zero-Waste’ Flight


Qantas Airlines of Australia has launched the world’s first-ever “zero-waste” flight. The flight was from Sydney to Adelaide.
All products and packaging on the plane will either reused, recycled or disposed of as compost.

Ans17-(b) Year of Next of Kin

Indian Army to Observe 2019 as ‘Year of Next of Kin’


The Indian Army has announced to commemorate the year 2019 as the ‘Year of Next of Kin’.
It plans to reach out to the next of kin of battle casualties, ex-servicemen and serving soldiers to inform them of entitled financial
benefits, welfare schemes and help resolve pension related problems since most of them are unaware of the financial benefits that are
entitled to them.
The Directorate of Indian Army Veterans will be the nodal agency to implement the initiative.

Ans18-(d) Bharatpe

Bharatpe launches India’s first UPI Bahi Khata for merchants.


BharatPe – India’s first Fintech start-up enabling payments for merchants through interoperable UPI QR codes, has announced its foray
into merchant services with a new app. The app allows merchants to record their cash/credit (Udhar) sales customer wise, request
accounts receivable from customers via SMS payment links, and keep track of accounts payable to suppliers, among other services.
The BharatPe app also serves as a networking platform for merchants to connect with other BharatPe merchants near them and to
collaborate with them to both sources and expand their businesses.

Page 62
STUDY IQ

Вам также может понравиться